Sunteți pe pagina 1din 32

1.

A middle aged lady with Rt sided thyroid gland


underwent hemithyroidectomy of the right side. After
the surgery, she had weak voice. Laryngoscopy was
done. There is loss of tension of the right vocal cord
and the right cord was flapping with respiration
.Laryngoscopy revealed the below finding.Which of the
following probably happened-

A. Rt recurrent laryngeal N. was damaged
B. Rt superior laryngeal N. was damaged
C. Both the nerves were damaged
D. This is a normal finding after hermithyroidectomy

2. An adult female patient has a history of foul smelling
ear discharge from the right side for the past 8
months. She now has fever for the last 3 days. The
fever is irregular associated with chills and rigor,
patient is also complaining of headache. Which of the
following is the most probable complication-
A. Labryrinthine fistula
B. Lateral sinus thrombophlebitis
C. Labyrinthitis
D. Temporal lobe abscess

3. A young man met with an accident leading to loss of
hearing in the right ear. On otoscopy examination the
tympanic membrane was intact .Pure tone audiometry
showed AB gap of 55 dB on the right ear with normal
cochlear reserve, which of the following will be
probable tympanometry finding-
A. As type tympanogram
B. Ad type tympanogram
C. B type tympanogram
D. C type tympanogram

4. Type V curve of Bekesy audiometry is seen in-
A. Retrocochlear lesion
B. Cochlear lesion
C. Malingerers
D. Normal hearers

5. A 35 year old man with unsafe
CSOM(cholesteatoma)was operated one year back.The
surgery done was MRM (Intact canal wall
mastoidectomy).He has recurrence of the disease
(cholesteatoma what would be the most imp. Step):
A. Antibiotics and close follow up
B. Radical mastoidectomy
C. Canal wall down MRM
D. Cortical mastoidectomy

6. A 3 year old child presents with fever, sore throat
and has difficulty in breathing. His radiograph is given
below. What is the most likely diagnosis?


A. TB larynx
B. Croup
C. Acute Epiglottitis
D. Laryngomalacia

7. A 6 year old child complains of hearing loss in both
ears with sleeping with open mouth. He had retracted
and bluish TM in both of the ears. You have planned to
do surgery. Which of the following would you not do
pre-operatively-
A. Culture and sensitivity of fluid aspirated from middle
ear
B. Impedance audiometry
C. X-ray of naso-pharynx->adenoid view
D. All should be done

8. Identify the instrument:


A. Lucs forceps
B. Mollisons mastoid retractor
C. Eves Tonsil Snare
D. Walsham forceps

9. In Auditory brainstem response tracing wave III
corresponds to



A. Cochlear nucleus
B. Superior olivary complex
C. Lateral lemniscus
D. Inferior colliculus

10. Unilateral pain in the ear in the absence of ear
disease can be due to all except:
A. Allergic rhinitis
B. Peritonsillar abscess
C. Cancer of the pyriform fossa
D. Temporomandibular joint dysfunction

11. A young boy of 18 years gets recurrent busts of
otitis externa after swimming. Which of the following
methods should be recommended?
A. Not to swim
B. Use 2% acetic acid after swimming
C. Use ear drops containing an antibiotic and steroid
D. Use an antifungal ear drops after swimming

12. Which of the following conditions predispose to
otitis media with effusion?
A. Cleft palate
B. Down's syndrome
C. Carcinoma nasopharynx
D. All the above

13. A 35 yr. male presents with the only c/o
hoarseness of voice for the past 2 yrs. He has been a
chronic smoker for 30 yrs, o/e a reddish area of
mucosal irregularity overlying a portion of both cords
are seen. Management would include all except-
A. Cessation of smoking
B. Bilateral cordectomy
C. Microlaryngeal surgery for biopsy
D. Regular follow up

14. A 55 year old male presents will sudden SN hearing
loss (Rt) side with tinnitus on the same side.Which of
the following is the most imp. Investigation-
A. Gadolium MRI
B. CT scan
C. Ear examination under microscopy
D. None of the above

15. Which of the following is the commonest
presentation of septal hematoma-
A. Unilateral nasal obstruction
B. Bilateral obstruction
C. Unilateral nasal bleeding
D. Bilateral nasal bleeding

16. In superior orbital Fissure syndrome all of the
following cranial nerves may be involved except-
A. III
B. IV
C. V
D. VI

17. Which of these may be indirect laryngoscopic
findings in the patient of phonasthenia-
A. Elliptical space between vocal cords (Thyroaretenoid
involved)
B. Triangular space in posterior glottis (Inter-aretenoid
involved)
C. Key hole appearance of glottis (both muscle involved)
D. All of the above

18. The facial nerve is damaged at the 2nd genu in the
middle ear leading to facial nerve palsy on the right
side. Which of these will be the stapedial reflex finding
when sound stimulus is given on the right side-
A. Reflex will be absent on the ipsilateral side only
B. Reflex will be absent on the contralateral side only
C. Reflex will be absent on both the sides
D. Reflex will be present on both the sides

19. A 7-year-old child developed acute otitis media. He
was treated with antibiotics for 10 days. His pain and
fever subsided completely but still had conductive
hearing loss. Your next line of treatment is:
A. Give another course of a different antibiotic
B. Do a myringotomy and culture the middle ear fluid
C. Do a myringotomy and insert a grommet
D. Wait and watch for 3 months for fluid to drain
spontaneously

20. MC Governs Technique may be a life saving
technique during which of the following-
A. Difficult tracheostomy
B. Difficult intubation
C. B/L Choanal atresia
D. CSF rhinorrhea

21. During expiration, maximum amount of air is
passed through which of these meatus-
A. Interior meatus
B. Middle meatus
C. Superior meatus
D. Supreme meatus

22. Long-standing case of otitis media with effusion
can develop all of the following complications except:
A. Retraction pockets
B. Cholesteatoma
C. Ossicular fixation
D. Cholesterol granuloma

23. A 5-year-oId male child had .acute otitis being
treated with ear drops, oral antibiotics and analgesics.
Two weeks after he presented with a swelling over the
mastoid, pain in the ear with pulsatile ear discharge
and fever. Now treatment of this child would include
all except:
A. W antibiotics
B. Cortical mastoidectomy
C. Modified radical mastoidectomy
D. Analgesics

24. Which of these above at paranasal sinus in true-
A. In children ethmoidal sinusitis is more common
B. Polyps are most common in maxillary sines
C. PNS malignancy is more common in Ethmoidal sinus
D. Mucocele is more common in splenoid sinus

25. Which of these secretary glands is not supplied by
fibres from facial nerve-
A. Parotid gland
B. Lacrimal gland
C. Submandibular gland
D. Sublingual gland

26. A pregnant woman in third trimester complains of
hearing her own sounds. Examination of the ear shows
movements of tympanic membrane synchronous with
respiration and especially exaggerated when nostril
on the contralateral side is occluded. Your diagnosis is:
A. Eustachian tube obstruction
B. Patulous eustachian tube
C. Otitis media with effusion
D. Otosclerosis

27. Which of these nerves is closely related to
sphenoid sinus(Anatomically)-
A. Ophthalmic nerve
B. Optic nerve
C. Oculomotor nerve
D. Trochlear nerve

28. Regarding glomus tumour of middle ear which of
the following statements is not correct?
A. More common in men
B. Grows very slowly
C. Diagnostic biopsy is contraindicated
D. Multicentric origin

29. Bulge on the roof of ear is called-
A. Promontory
B. Arcuate eminence
C. Tegmen Tympan
D. None

30. A 45 year old person had Rt enlarged tonsil for 6
weeks. Tonsillectomy was done for this what should be
the next logical step-
A. Sent the specimen for histo-pathological Examination
B. Sent the specimen for culture sensitivity examination
C. Take a swab from tonsillar bed for culture sensitivity
examination
D. Styloid process should be removed

31. Which of these about nasal packing is true-
A. After nasal packing, patient always be deeply sedated
B. Only the bleeding nasal cavity should be packed
C. Both anterior and posterior nasal packing may be
done together on the patient
D. Nasal pack should be changed after every 24 hrs

32. Which of the following signs is not 8asoclated with
glomus tumour?
A. Rising sun appearance
B. Aquino's sign
C. Brown's sign
D. Griesinger's sign

33. Contributing arches for the development of pinna-
A. First arch
B. First arch and second arch
C. Second and third arch
D. Fourth arch

34. External auditory canal is supplied by all except-
A. Trigeminal
B. Vagus
C. Facial
D. Glossopharyngeal

35. The external landmark for mastoid antrum is-
A. Concha
B. Scaphoid fossa
C. Cymba concha
D. Triangular fossa

36. Acute masoiditis will cause the following-
A. Tragnal sign positive
B. Tenderness at Cymba conchae
C. Both
D. None

37. Tympanic membrane attains it angulation of 55*
at-
A. Birth
B. 4-5 years
C. 5-6 years
D. 7 years

38. Treatment of choice for a glomus tumour
restricted to promontory of middle ear is:
A. Surgical removal
B. Embolisation
C. Radiation therapy
D. Wait and watch

39. Thickness of lateral wall of mastoid antrum at
birth-
A. 1 mm
B. 2 mm
C. 4 mm
D. 5 mm

40. Eustachian tube opens-
A. Above the posterior end of the inferior turbinate
B. Below the inferior turbinate
C. Below and behind the inferior terbinate
D. Behind the middle turbinate

41. Apex of the cocheal perceives-
A. Low frequency sounds
B. High frequency sounds
C. Both
D. None

42. Which of the following is not the site of
paraganglioma?
A. Carotid bifurcation
B. Jugular foramen
C. Promontory of middle ear
D. Geniculate ganglion

43. All are approaches to petrous apex except-
A. Ramadiers
B. Rosens
C. Almoors
D. Frenckners

44. Commonest site for cholesteatoma-
A. Posterior mesotympanum
B. Prussaks space
C. Anterior epitympanum
D. Protympanum

45. Positive fistula sign is seen-
A. Otosclerosis
B. Menieres disease
C. C.S.O.M
D. Dead labyrinth

46. Browns sign seen is-
A. Acoustic neuroma
B. Glomusjugulare
C. Congenital cholestroma
D. Meningioma

47. A 38 year old man had labyrinthine fistula with
positive fistula sign.Two weeks later fistula sign
became negative without any treatment.Which of the
following will be the rinnes test(tunning fork) finding-
A. True positive rinnes test
B. False positive rinnes test
C. True negative rinnes test
D. False negative rinnes test

48. A male presents with dull TM, deafness and
tinnitus, type B curve impedence and neck mass, the
most likely cause is-
A. Canasopharynx
B. Ca middle ear
C. TB of neck nodes
D. Cholesteatoma



49. CROS hearing aids is used for-
A. Bilateral severe deafness
B. Severe unilateral deafness
C. Mild neural deafness
D. None of the above

50. Which of the following is true about the
complications of acute mastoiditis except-
A. The most common abscess is post/retro aural abscess
B. Lucs abscess is in the superior canal wall
C. Bezolds abscess is usually subperiosteal
D. Zygomatic abscess is usually subperiosteal

51. Lucs abscess is seen in-
A. Behind the mastoid
B. In the external auditory canal
C. In the sheath of sternocleidomastoid
D. None of the above

52. Treatment of stag IV carcinoma larynx is-
A. Surgery
B. Chemotherapy
C. Surgery + Radiotherapy
D. Surgery Chemotherapy

53. Early vocal nodules are best treated by-
A. Microlaryngeal surgery
B. Laser excision
C. Voice rest
D. Speech therapy

54. Most common fungal infection of the paranasal
sinus-
A. Rhinosporidiosis
B. Mucormycosis
C. Aspergillosis
D. Monilia

55. Aim of mastoid surgery In CSOM which should
receive first priority is:
A. Making the ear dry
B. Improvement in hearing
C. Preservation of hearing
D. Rendering the ear safe

56. Hypoactive labyrinth cause nystagmus to-
A. Same side
B. Opposite side
C. Rotatory nystagmus
D. No-nystagmus

57. Vestibular schwannoma 1 cm in diameter, the
investigation of choice is-
A. CT scan
B. MRI
C. Angiography
D. Plain X-ray

58. A 30-year-old woman with family history of
hearing loss from her mother's side developed hearing
problem during pregnancy. Hearing loss is bilateral
slowly progressive, with bilateral tinnitus that bothers
her at night. Pure tone audiometry shows conductive
hearing loss with an apparent bone conduction
hearing loss at 2000 Hz. What is the most likely
diagnosis?
A. Otosclerosis
B. Acoustic neuroma
C. Otitis media with effusion
D. Sigmoid sinus thrombosis

59. The prevalence of clinical otosclerosis is highest In:
A. Japanese
B. Caucasians
C. Americans
D. Africans

60. Accessory ostia in the lateral wall are located in
the-
A. Middle meatus
B. Posterior fontanelle
C. Inferior meatus
D. None of the above

61. 45 year old diabetic patient presents with the
history of black necrotic feling up the nasal cavity and
diplopia of the affected side,the treatment of choice is-
A. FESS
B. Endoscopic clearance
C. Local debridement
D. Local debridement with orbital exenteration

62. Which of the following tracheostomy tubes have
double cuffs-
A. Shileys tube
B. Salpekar tube
C. Montgomery tube
D. Keonigs tube

63. Otosclerosis is:
A. Autosomal dominant
B. Autosomal recessive
C. X-linked disease
D. Mitochondrial disorder

64. Extension of angiofibroma to which of these areas
may require pre-operative radiotherapy-
A. Intra cranial extension
B. Nasal extension
C. Pterygopalaine fossa extension
D. None of the above

65. Which of the following is not a part of temporal
bone-
A. Squamous bone
B. Occipital bone
C. Petrous bone
D. Tympanic plate

66. Most common site of otosclerosis is-
A. Oval window
B. Round window
C. Fissula ante fenestram
D. None

67. Most common site for Initiation of stapedial
otosclerosis is:
A. Fissula ante-fenestram
B. Fossula post-fenestram
C. Footplate of stapes
D. Margins of stapes

68. Endoscopic sinus surgery is used for which of the
following-
A. Antrochoanal polyp removal
B. Repair of CSF rhinorrhea
C. Optic nerve decompression
D. All

69. Drug of choice for allergic fungal sinusitis is-
A. I/U amphotericin B
B. Antibiotics
C. Corticosteroids
D. None

70. A 10-yr old boy is having sensory neural deafness.
He showed no improvement with conventional
hearing aids. Most appropriate management is-
A. Bone conduction hearing aids
B. Fenestration
C. Stapes fixation
D. Cochlear implant

71. The following is difficult to see during indirect
laryngoscopy-
A. True cords
B. False cord
C. Posterior commissure
D. Anterior commissure

72. Pneumatized middle turbinate is called-
A. Bulla ethmoidalis
B. Concha bullosa
C. Onodi cell
D. Heller cell

73. Genetic defect leading to complete failure of inner
ear development is of the following type-
A. Michel aplasia
B. Mondini aplasia
C. Scheibe aplasia
D. Alexander aplasia

74. In the pathogenesis of otosclerosis, the disease
process starts in:
A. Periosteal layer of otic capsule
B. Endosteal layer of otic capsule
C. "Bone of otic capsule which develops from cartilage
D. Mucoperiosteum of the promontory

75. Which of the following statements Is not true
about stapedial otosclerosis?
A. Slowly progressive conductive hearing loss with
normal tympanic membrane
B. Rinne's test is negative
C. Tympanogram is Ao type
D. Eustachian tube is patent

76. Which of the following type of tympanoplasty is
called columellatympanoplasty-
A. Type I
B. Type II
C. Type III
D. type IV

77. Which of the following is the newer device in the
treatment of menieres disease-
A. P-100
B. mienettes device
C. Both
D. None

78. Schwartze's sign is:
A. Swelling over the mastoid
B. Reddish hue seen in the hypotympanum behind an
intact tympanic membrane
C. Improved hearing in noisy surroundings
D. Reddish hue seen over the promontory

79. A 45 years old lady presented with a swelling in the
parotid gland for the past 6 months .FNAC done from
this area revealed lympho-epithelioma.All are true of
lympho-epithelioma except-
A. Parotid gland is the most common site of lympho-
epithelioma in the head and neck region
B. It is known to be caused by EBV
C. The epithelial cells are mainly malignant with
lymphoid tissue
D. It is a highly radio sensitive tumour

80. Singular nerve supplies which of the following
semicircular canals-
A. Superior canal
B. Posterior canal
C. Lateral canal
D. There is no such nerve in the inner ear

81. A 30 year old lady with conductive hearing loss on
atoscopy tympanic membrane has flamingo pink
blush. Impedance audiometry shows AS curve the
treatment of choice is-
A. NaF
B. Stapedectomy
C. Fenestration surgery
D. Wait and watch

82. Which of the following is seen in posterior
rhinoscopy-
A. Vomer
B. Inferior turbinate
C. Eustachian tube opening
D. All

83. Holman miller sign is seen in-
A. Angiofibroma
B. Nasopharyngeal Ca
C. Antrochoneal polyp
D. All

84. In an otosclerotic patient Rinne's test was negative
with a tuning fork of 512 Hz but positive for 1024 Hz.
His minimum predicted AB gap on audiometry would
be:
A. 20dB
B. 30dB
C. 45 dB
D. 60 dB

85. The triad constituting syndrome of Van der Hoeve
Includes all except:
A. Osteogenesis imperfecta
B. Conductive hearing loss
C. Blue sclera
D. Preauricular sinuses

86. Carhart's notch In audiogram Is deepest at the
frequency of:
A. 0.5 kHz
B. 2.0 kHz
C. 4.0 kHz
D. 8.0 kHz

87. All of the following statements are true about
Carhart's notch except:
A. It is a sensorineural hearing loss
B. Maximum loss is centred at 2 kHz
C. Seen only in stapes fixation
D. Cannot by reversed

88. Treatment of choice for otosclerosis In a young
person employed In office is:
A. Hearing aid
B. Stapedectomy
C. Stapes mobilisation only
D. Fenestration operation

89. Which ossicle is most commonly damaged by
cholesteatoma-
A. Malleus
B. Incus
C. Stapes
D. All have equal percentage

90. Condition in which loud sounds produce giddiness
is called:
A. Paracusis Willisi
B. Hennebert's sign
C. Tullio phenomenon
D. Otolithic crisis of Tumarkin

91. A 31-year-old female patient complains of bilateral
Impairment of hearing for the past 5 years. On
examination, tympanic membrane (TM) is normal and
audiogram shows a bilateral conductive loss.
Impedance audiometry shows AS type of curve and
absent acoustic reflex. All constitute part of treatment
except:
A. Hearing aid
B. Stapedectomy
C. Sodium fluoride
D. Gentamicin therapy

92. During otoscopy of patients right ear otoscopy
should ideally be held in which hand-
A. Right hand
B. Left hand
C. Either
D. None

93. Which of the following lesions is pre-cancerous-
A. Keratosis of tonsil
B. Keratosis of larynx
C. Aphthous ulcers
D. All

94. A 31-year-old female patient complains of bilateral
Impairment of hearing for the past 5 years. On
examination, tympanic membrane (TM) is normal and
audiogram shows a bilateral conductive loss.
Impedance audiometry shows AS type of curve and
absent acoustic reflex. All constitute part of treatment
except:
A. Hearing aid
B. Stapedectomy
C. Sodium fluoride
D. Gentamicin therapy

95. Most common cause for bilateral conductive
deafness in a child is:
A. Otosclerosis
B. Otitis media with effusion (OME)
C. Acute otitis media
D. Congenital cholesteatoma

96. Ramavati is 40 years old female presented with a
progressively increasing lump in the parotid region.
On oral examinations, the tonsil was pushed medially.
Biopsy showed it to be pleomorphic adenoma. The
appropriate treatment is-
A. Superficial parotidectomy
B. Lumpectomy
C. Conservative total parotidectomy
D. Enucleation

97. Which of the following is not an indication of
radiotherapy in Pleomorphic adenoma of parotid-
A. Involvement of deep lobe
B. 2nd histologically benign recurrence
C. Microscopically positive margins
D. Malignant transformation

98. Nerve which lies in association to Wartons duct is-
A. Hypoglossal
B. Lingual
C. Lingual nerve
D. Mandibular branch of facial nerve

99. A 55-year-old female presents with tinnitus,
dizziness and history of progressive deafness.
Differential diagnosis Includes all except:
A. Acoustic neuroma
B. Endolymphatic hydrops
C. Meningioma
D. Histiocytosis X

100. Bilateral destruction of auditory cortex leads to-
A. Total deafness
B. Hearing defect for higher frequency sounds
C. Deficiency in interpretation of sounds
D. Inability for orientation of sounds

101. Tympanic membrane develops from:
A. Ectoderm
B. Mesoderm
C. Endoderm
D. All the three germinal layers

102. Number of centres from which bony labyrinth
ossifies are:
A. 6
B. 8
C. 10
D. 14

103. Nerve supply of tympanic membrane is derived
from all except
A. Auriculotemporal nerve
B. Auricular branch of vagus
C. Great auricular nerve
D. Glossopharyngeal nerve

104. Area of stapes footplate is:
A. 1.5 sq mm
B. 2.2 sq mm
C. 3.0 sq mm
D. 3.2 sq mm

105. The pinna attains 90-95% of adult size by:
A. Birth
B. 5-6 years
C. 9-10 years
D. 11-12 years

106. Sensory nerve supply of the middle ear comes
from:
A. Vagus nerve
B. Caroticotympanic nerves
C. Glossopharyngeal nerves
D. Chorda tympani

107. Citelli's angle is:
A. Solid angle
B. Cerebellopontine angle
C. Sinodural angle
D. Genu of facial nerve

108. In a normal ear, which of the following is a false
statement?
A. Total length of external ear canal is 36 mm
B. Pinna has to be pulled upwards and backwards to see
the tympanic membrane
C. External ear canal does not contain ceruminous glands
or hair follicles in the deep bony part
D. Dehiscences may be seen in outer cartilaginous canal

109. Which of the following statements is true?
A. Korner's septum in the mastoid separates squamous
cells from the deeper petrosal cells
B. Facial recess lies medial to the sinus tympani
C. Trautmann's triangle forms an important surgical
landmark to locate endolymphatic sac
D. Arcuate eminence is landmark for lateral semicircular
canal

110. Which of the following is not true about the
appearance of tympanic membrane?
A. A red tympanic membrane may be normal in a crying
child
B. A retracted tympanic membrane shows prominent
lateral process of malleus and foreshortened handle of
malleus
C. A bulging tympanic membrane loses all landmarks
D. Creation of positive or negative pressure with Siegel's
speculum or pneumatic otoscope has no effect
onmovement of normal tympanic membrane

111. During superficial parotidectomy, the most
reliable landmark to Identify main trunk of facial
nerve is:
A. Mastoid tip
B. Styloid process
C. Tympanomastoid suture
D. Cartilage of external auditory canal

112. Endolymph is formed in:
A. Utricle
B. Endolymphatic sac
C. Scala media
D. Scala tympani

113. Communication between middle ear and
eustachian tube is obliterated surgically in:
A. Cortical mastoidectomy
B. Modified radical mastoidectomy
C. Radical mastoidectomy
D. Bondy's mastoidectomy

114. Stapes superstructure develops from:
A. Meckel's cartilage
B. Reichert's cartilage
C. Both (a) and (b)
D. Both (a) and (b) plus bony otic capsule

115. Interpretation of SERA (brain-stem evoked
response audiometry) is affected by:
A. Age of the child
B. Sex of the child
C. Sedation
D. Sleep

116. Inner ear malformation in fetus can occur when
mother during pregnancy is exposed to all except:
A. Radiation
B. German measles
C. Cytomegalovirus
D. Lithium


117. Cause of sensorineural hearing loss due to
furosemide toxicity is:
A. Damage to outer cells of cochlea
B. Damage to inner cells of cochlea
C. Stria vascularis
D. Cochlear nerve

118. All are true about Jacobson's nerve except:
A. It is a branch of superior ganglion of vagus
B. Supplies middle ear and mastoid air cells
C. Supplies secretomotor fibres to parotid
D. Section of this nerve relieves gustatory sweating

119. In a sitting position with head tilted 60
0

backward, which of the following canals is stimulated
during caloric testing:
A. Superior
B. Posterior
C. Lateral
D. Both lateral and superior

120. Dorello's canal transmits which of the following
nerves?
A. Ophthalmic division of CN V
B. Abducens (CN VI)
C. Facial nerve (CN VII)
D. Statoacoustic nerve (CN VIII)

121. Costen's syndrome is characterised by all except:
A. Otalgia
B. Vertigo
C. Tinnitus
D. Recurrent dislocation of temporomandibular joint

122. Operation of choice for coalescent mastoiditis is:
A. Cortical mastoidectomy
B. Modified radical mastoidectomy
C. Radical mastoidectomy
D. Fenestration operation

123. The cough response caused while cleaning the ear
canal is mediated by stimulation of:
A. The V cranial nerve
B. Innervation of external ear canal by C1 and C2
C. The X cranial nerve
D. Branches of the VII cranial nerve

124. A 38-year-old gentleman reports of decreased
hearing in the right ear for the last 2 years. On testing
with a 512-Hz tuning fork, the Rinne's test (without
masking) is negative on the right ear and positive on
the left ear. With the Weber's test the tone is perceived
louder in the left ear. The patient most likely has:
A. Right conductive hearing loss
B. Right sensorineural hearing loss
C. Left sensorineural hearing loss
D. Left conductive hearing loss

125. Which of the following Is not a typical feature of
Meniere's disease?
A. Sensorineural deafness
B. Pulsatile tinnitus
C. Vertigo
D. Fluctuating deafness

126. Which of the following is not a typical feature of
malignant otitis externa?
A. Caused by Pseudomonas aeruginosa
B. Patients are usually old
C. Mitotic figures are high
D. Patient is immune compromised

127. Treatment of choice for glue ear is:
A. Conservative
B. Myringotomy with cold knife
C. Myringotomy with ventilation tube insertion
D. Myringotomy with diode laser

128. Which of Ute following is true regarding facial
nerve palsy associated with temporal bone fracture?
A. Common with longitudinal fracture
B. Common with transverse fracture
C. Always associated with CSF otorrhoea
D. Facial nerve injury is always complete

129. The posterosuperior retraction pocket, if allowed
to progress, will lead to:
A. Sensorineural hearing loss
B. Secondary cholesteatoma
C. Tympanosclerosis
D. Tertiary cholesteatoma

130. A 30-year-old male is having attic cholesteatoma
of left ear with lateral sinus thrombophlebitis. Which
of the following will be the operation of choice?
A. An intact canal wall mastoidectomy
B. Simple mastoidectomy with tympanoplasty
C. Canal wall down mastoidectomy
D. Mastoidectomy with cavity obliteration

131. Which is the Investigation of choice in assessing
hearing loss in neonates?
A. Impedance audiometry
B. Brain-stem evoked response audiometry
C. Free field audiometry
D. Behavioural audiometry

132. Which of the following conditions causes the
maximum hearing loss?
A. Ossicular disruption with intact tympanic membrane
B. Disruption of malleus and incus as well tympanic
membrane
C. Partial fixation of the stapes footplate
D. Otitis media with effusion

133. Use of Siegel's speculum during examination of
the ear provides all except:
A. Magnification
B. Assessment of movement of the tympanic membrane
C. Removal of foreign body from the ear
D. As applicator for the powdered antibiotic to ear

134. All are true for Gradenigo's syndrome except:
A. It is associated with conductive hearing loss
B. It is caused by an abscess in the petrous apex
C. It leads to involvement of the cranial nerves V and VI
D. It is characterized by retro-orbital pain

135. In right middle ear pathology, Weber's test will
be:
A. Normal
B. Centralized
C. Lateralised to right side
D. Lateralised to left side

136. A 25-year-old woman suffering from bilateral
hearing loss for 6 years which became profound with
pregnancy. On tympanogram, which of the following
curve is obtained?
A. AD
B. As
C. B
D. A

137. Unit of Intensity while testing for threshold of
hearing in an audiogram is:
A. dB SL
B. dB HL
C. dB A
D. dB SPL
E. dBnHL

138. Otoacoustic emissions are produced by:
A. Inner hair cells
B. Outer hair cells
C. Basilar membrane
D. Auditory nerve

139. Speech frequencies Include:
A. 125 250 500 Hz
B. 250 500 1000 Hz
C. 500 1000 2000 Hz
D. 1000 2000 3000 Hz

140. A young female patient with long history of
sinusitis presents with frequent fever along with
personality changes & headache of recent origin. The
fundus examination revealed papilledema. The most
likely diagnosis is :
A. Frontal lobe abscess
B. Meningitis
C. Encephalitis
D. Frontal bone osteomyelitis

141. Majority of cerebellopontine angle tumors are:
A. Acoustic neuroma
B. Meningioma
C. Haemangioma
D. Pontine Glioma

141. Nasopharyngeal angiofibroma arises from
A. Posterolateral part of nasopharynx
B. Vault of nasopharynx
C. Posterior part of nasal cavity
D. Pterygomaxillary fossa

142. Anterior ethmoidal sinus opens into
A. Middle meatus
B. Anterior turbinate
C. Superior meatus
D. Inferior meatus

143. Treatment of complete URT obstruction is
A. Cricothyroidotomy
B. Nasal intubation
C. Needle in trachea
D. Endotrachial intubation

144. Treatment of bilateral abductor vocal cord palsy
is
A. Adrenaline
B. Tracheostomy
C. Endotracheal intubation
D. None of the above

145. Quantity of endolymph is
A.
B.
C.
D.

146. Korner's septum is formed by
A. Petro-squamous suture
B. Petro-squamous fissure
C. Lateral margin of tegmen tympani
D. Squamous tympanic fissure

147. External auditory canal is formed
A. 1st branchial arch
B. 2nd branchial cleft
C. 1st branchial cleft
D. 1st visceral cleft

148. After Type 1lI tympanoplasty, there is loss of all
EXCEPT
A. 15 db
B. 25 db
C. 30 db
D. 5 db

149. After facial canal short process of incus, 3rd
landmark for posterior tympanoplasty is
A. Tympanic annulus
B. Mastoid process
C. Zygomatic ridge
D. None of the above

150. Glomus jugular tumour is seen in
A. Caroticotympanic
B. Anterior tympanic membrane
C. Hypotympanum
D. Roof

151. Schwartze sign is seen in
A. Otosclerosis
B. Meniere's disease
C. Glomustumour
D. Glomus Jugulare

152. All are features of tympanic membrane rupture
(printed oesophageal rupture in paper) EXCEPT
A. Tinnitus
B. Vertigo
C. Conductive deafness
D. Fullness in ear

153. Treatment of choice in atticoantral perforation is
A. Surgery
B. Medical treatment
C. Medical & surgical treatment
D. Radiotherapy

154. Rhinosporodiosis is caused by
A. Protozoal infection
B. Viral infection
C. Bacterial infection
D. Fungal infection

155. Treatment of choice in antrochoanal polyp is
A. Surgery
B. Medical
C. Medical & surgical treatment
D. Wait and watch

156. Eustachian tube opens into middle ear cavity at
A. Anterior walls
B. Hypotympanum
C. Superior surface
D. Posterior wall

157. Hyperacusis in belle's palsy is due to
A. Stapedial paralysis
B. Tensor tympani paralysis
C. Tensor palati paralysis
D. Trigeminal nerve paralysis

158. Cochlear implant used in
A. Sensorineural deafness not improving with medical
therapy
B. Otosclerosis
C. Meniere's disease
D. Glomustumour

159. TRUE about Ramsay - hunt syndrome EXCEPT
A. Involves VII nerve
B. May involves VIII nerve
C. Surgical removal gives excellent prognosis
D. Causative agent is virus

160. Earliest manifestation of cerebellopontine angle
tumor
A. Loss of corneal reflex
B. Ipsilateral lateral squint
C. Ipsilateral tongue paralysis
D. Ipsilateral pupillary dilation

161. Most common cause of stridor in neonates is
A. Foreign body aspiration
B. Laryngornalacia
C. Pneumonia
D. Epiglottitis

162. Most common cause of unilateral hearing loss is
A. Measles
B. Mumps
C. Chickenpox
D. Rubella

163. Most common extra-cranial complication of ASOM
is
A. Facial nerve paralysis
B. Lateral sinus thrombosis
C. Sub periosteal abscess
D. Brain abscess

164. Rhinophyloma occurs due to
A. Hypertrophy of meibobian gland
B. Hypertrophy of the sebaceous gland
C. Malignant tumour
D. Papulo-pearly nodule

165. Hypertrophy of lingual tonsil occurs in
A. Tonsillolith
B. Tonsillectomy patient
C. Tonsillar cyst
D. Peritonsillar abscess

166. Which of the following is known as fourth
turbinate
A. Posterior ethmoid cells
B. Anterior ethmoid cells (Aggar-nasi)
C. Medial ethmoid cells
D. Lateral ethmoid cells

167. Ciliary movement rate of nasal mucosa is
A. 1-2 mm/min
B. 2-5 mm/min
C. 5-10 mm/min
D. 10-12 mm/min

168. Most common sequence of ASOM is children
A. Deafness
B. Chronic-mastoiditis
C. Cholestetoma
D. Labyrinthitis

169. Which of the following part NOT included in
hypopharynx is
A. Pyriform sinus
B. Post cricoid region
C. Anterior pharyngeal wall
D. Posterior pharyngeal wall

170. Which of the following region is involved in
plummer-vinson syndrome
A. Pyriform sinus
B. Post cricoid region
C. Valleculae
D. Posterior pharyngeal wall

171. Motion sickness can be prevented by all EXCEPT
A. Meteclopramide
B. Domperidone
C. Astimazole
D. Cisapride

172. Semicircular canal involved in positional vertigo
is
A. Lateral
B. Superior
C. Inferior
D. Posterior

173. Rinnes test will be negative if loss of hearing is
A. 0-15dB
B. 15-20dB
C. 25-45dB
D. 45-60dB

174. Lever ratio of tympanic membrane is
A. 1.4-1
B. 1.3-1
C. 18.2-1
D. 1.5-1

175. All are tunning fork test EXCEPT
A. Schwaback test
B. Grants test
C. Rinne's test
D. Weber's test

176. Unilateral past pointing nystagmus is seen in
A. Paralysis of posterior semcircular canal
B. Paralysis of medial semicircular canal
C. Follicular node lesion
D. Cerebral hemisphere lesion

177. After sore throat patient develop acute sinusitis
presents with all EXCEPT
A. Diplopia
B. Rhinorrhoea
C. Headache
D. Foul smelling

178. Allergic fungal sinusitis, true are all EXCEPT
A. Tissue invasion is seen
B. Endoscopic sinus treatment
C. Enhancement C.T. scan is done
D. Steroids are useful

179. Nasopharynx carcinoma is
A. Basal cell carcinoma
B. Squamous cell carcinoma
C. Adenocarcinoma
D. Epidermoid carcinoma

180. Radical neck dissection is done in all EXCEPT
A. Supraglottic CA
B. Glottic CA
C. Transglottic CA
D. Pyriform fossa tumour

181. 12 years child presented with discharge ear with
deafness, on examination. Cholesteatoma &
sensorineural deafness is present. Treatment of choice
is
A. Radical mastoidectomy
B. Modified radical mostoidectomy
C. Medical treatment
D. Palliative surgery

182. In acoustic neuroma, cranial nerve to be involved
earliest is:
A. V
B. VII
C. X
D. IX

183. A post dental treatment, presented with pain &
swelling of SCM, examination reveals medially shift of
the tonsil, Diagnosis is
A. Parapharyngeal abscess
B. Retrophoryngeal abscess
C. Ludwing's angina
D. Dental cyst

184. A child presented with biphasic stridor,
treatment includes all EXCEPT
A. Steroids
B. Oxygen
C. Radiotherapy
D. Tracheostomy

185. "Pachydermia laryngitis" commonest site
involvement is
A. Inter arytenoid fold
B. Posterior 1/3 & anterior 1/3 commisure
C. Anterior 1/3 commissure
D. Vestibular fold

186. Which of the following would be the most
appropriate treatment for rehabilitation of a patient
who has bilateral profound deafness following surgery
for bilateral acoustic schwanoma:
A. Bilateral high powered digital hearing AID
B. Bilateral cochlear implants
C. Unilateral cochlear implant
D. Brain stem implant

187. True about CP angle tumour is:
A. Absent corneal reflex
B. Ipsilateral rectus palsy
C. Pupillary dilatation
D. Medial rectus palsy

188. Antro-choanal polyp are usually:
A. Single and unilateral
B. Multiple and bilateral
C. Multiple and unilateral
D. Single and bilateral

189. Main vascular supply of Little's area in all, Except:
A. Septal branch of superior labial artery
B. Nasal branch of sphenopalatine artery
C. Anterior ethmoidal artery
D. Palatal branch of sphenopalatine

190. Nasopharyngeal carcinoma presents as:
A. Epistaxis
B. Mass
C. Headache
D. Vertigo

191. White patch in the throat may be due to:
A. Streptococcus
B. Actinomycetes
C. Sporotrichosis
D. All of these

192. Injury to superior laryngeal nerve causes:
A. Hoarseness
B. Paralysis of vocal cords
C. No effect
D. Loss of timbre of voice

193. Most common location of vocal nodule:
A. Anterior 1/3 and posterior 2/3 junction
B. Anterior commisure
C. Posterior 1/3 and anterior 2/3 junction
D. Posterior commissure

194. A 31 year old female patient complains of
bilateral impairment of hearing for the past 5 years.
On examination tympanic membrane is normal and
audiogram shows a bilateral conductive deafness.
Acoustic reflexes are absent. All constitute part of
treatment, except
A. Hearing aid
B. Stapedectomy
C. Sodium fluoride
D. Gentamycin

195. Commonest complication of CSOM is:
A. Conductive deafness
B. Meningitis
C. Temporal lobe abscess
D. Cholesteatoma

196. Patient presents with high fever, Signs of raised
ICT and a past history of chronic otitis media likely
diagnosis is:
A. Brain abscess
B. Pyogenic meningitis
C. Acute subarachnoid hemorrhage
D. Acute osteomyelitis of skull bone

197. The part most commonly involved in otosclerosis
is:
A. Oval window
B. Round window
C. Tympanic membranes
D. Malleus

198. The most common cause of laryngeal stridor in a
60 years old:
A. Nasopharyngeal carcinoma
B. Thyroid carcinoma
C. Foreign body aspiration
D. Carcinoma larynx

199. Thudichum's nasal speculum is used to visualize:
A. Anterior nasal cavity
B. Posterior nares
C. Tonsils
D. Larynx

200. Pulsatile Tinnitus in ear is due to:
A. Malignant otitis modes
B. Osteoma
C. Mastoid reservoir
D. Glomus jugular tumour
1.Answer is B.
Unilateral damage-->present with voice change and
hoarseness
Bilateral-->breathing difficulties and aphonia
Rt. recurrent laryngeal n. d/t relative medial location is
more susceptible to damage during thyroid Sx
M/C cause of unilateral vocal cord palsy-
Idiopathic
Thyroid Sx
M/C cause of bilateral vocal cord palsy- Thyroidectomy
M/C cause of recurrent laryngeal nerve paralysis--
>bronchogenic carcinoma
Superior Laryngeal Nerve Palsy
Noticeable deviation of posterior commissure to paralyzed
side during phonatory effort
At rest, the vocal fold on paralyzed side is slightly
shortened and bowed, and may be depressed below level
of normal side. Loss of sensation to the supraglottic larynx
can cause subtle symptoms such as frequent throat
clearing, paroxysmal coughing, voice fatigue, vague foreign
body sensations.
Loss of motor function to cricothyroid muscle can cause a
slight voice change, which the patient usually interprets as
hoarseness. Most common finding is diplophonia (with
decreased range of pitch, most noticeable when trying to
sing

2. Answer is B.
Catch-->irregular(other findings may be found in abscess
also) .

Clinical features vary according to the stage of the disease.
Patients present with headache, fever, and otorrhea. The
classic case of lateral sinus thrombosis in the preantibiotic
era typically produced a picket fence fever curve, due to
the periodic release of hemolytic streptococci from the
septic sinus thrombus. With the occlusion of the lumen of
the sinus, an interruption of the cortical venous circulation
results in headache, papilledema, and increased
intracranial pressure. Involvement of the torcular and
sagittal sinus can result in otitic hydrocephalus.

Tenderness and edema over the mastoid (the Griesinger
sign) are highly suggestive of lateral sinus thrombosis and
reflex thrombosis of mastoid emissary vein. With the
extension of thrombophlebitis into the jugular bulb and
internal jugular vein, pain may be present in the neck,
particularly on rotation. Internal jugular vein may be
palpated in the neck as a tender cord. The 9th, 10th, and
11th cranial nerve may be paralyzed by the presence and
pressure of a clot in the jugular bulb (jugular foramen
syndrome).

Because the right transverse is usually dominant, the
symptoms are more likely to occur when this sinus is
involved. Recovery depends on the development of
collateral circulation or possibly recanalization of the
sinus. Because of this, the presence of anastomotic channel
is important for recovery

3. Answer is B. Type A refers to eardrum movement
within normal limits. Type B indicates little or no eardrum
movement suggesting fluid in the middle ear space. A child
with this type of tympanogram needs medical attention.
Type C refers to a middle ear with negative pressure. Such
a tympanogram may be caused by retraction of the
eardrum or blockage of the Eustachian tube. Patients that
present with Type As tympanograms normally have a
history of middle ear problems ranging from childhood
otitis media, otosclerosis and tympanosclerosis. TYPE Ad
may present in patients who have had multiple
perforations in the past, or multiple sets of grommets.
Patients with his tympanogram may also have an
ossicular chain disruption. (Rx-Ossiculoplasty)

4.Answer is C.

This uses continuous and pulsed tone tracings. The normal
graph recorded may be interleaved / continuous tracings
below pulsed tone tracings. In patients with non-organic
hearing loss will have opposite curves - their pulsed
tracings are tracked below the continuous tracings. This
type of curve is known as Type V Bekesy pattern.

Patient himself does the audiometry
I- N or conductive hearing loss
II- Cochlear
III/IV-Retrocochlear
V-Malingers

5.Answer is C.
Remember the primary aim is to render the ear safe

6. Ans : B Croup is characterized by a "barking" cough,
stridor, hoarseness, and difficult in breathing which
usually worsens at night. The "barking" cough is often
described as resembling the call of a seal or sea lion. The
stridor is worsened by agitation or crying, and if it can be
heard at rest, it may indicate critical narrowing of the
airways. As croup worsens, stridor may decrease
considerably. In radiology, the steeple sign is a radiologic
sign found on a frontal neck radiograph where subglottic
tracheal narrowing produces an inverted "V" shape within
the trachea itself. The presence of the steeple sign
supports a diagnosis of croup


7.Answer is A.
Glue ear fluid is sterile. X-Ray of nasopharynx: Adenoid
view

8. Ans: C
9. Ans: B

This mnemonic helps when remembering the ascending
order of structures that corresponds to each waveform in
an auditory brainstem response (ABR) tracing:

E COLI
Mnemonic

E - eighth nerve action potential (wave I)
C - cochlear nucleus (wave II)
O - olivary complex (superior) (wave III)
L - laterallemniscus (wave IV)
I - inferior colliculus (wave V)

10. Answer A

Pain in the ear is referred from CN IX (peritonsillar
abscess). CN X (cancer of the pyriform fossa) and CN V
(ulcer tongue and temporomandibular joint).

11. Answer D
Use of 2% acetic acid or dilure alcohol is the most-effective
method to prevent otitis externa after swimming.
Antibiotic, antifungal or steroid containing drugs are not
recommended for prophylaxis.

12. Answer DAll the above
Otitis media with effusion (OME) occurs due to defective
eustachian tube function. All of the listed conditions affect
function of the lube and cause OME. In a unilateral OME in
an adult, always exclude carcinoma nasopharynx.

13.Answer is B.
Any patient in cancer age group having persistent or
gradual hoarseness of voice for >3 wks must have
laryngeal examination to exclude cancer
Clinical presentation and examination reveals a
premalignant lesion of the vocal cords
Rx:
a.Cessation of smoking
b.Micro laryngeal Sx for biopsy
c.Regular follow up
Best treatment at this stage is radiotherapy rather than
bilateral cordectomy

14. Answer is A.
Gold standard for acoustic neuroma-->gadolinium MRI
Unilateral glue ear(adult)-->d/t nasopharyngeal Ca

15. Answer is B.
M/C cause of septal abscess-->septal hematoma
Always drain a septal hematoma as septal perforation is
most common complication(since cartilage takes nutrition
from perichondrium)

16. Answer is C.
Superior orbital fissure syndrome- Infection of sphenoid
sinus may affect structures of superior orbital fissure.
Symptoms-
1. deep orbital pain
2. frontal headache
3. progressive paralysis of CN VI, III, IV in that order
Orbital apex syndrome= Orbital fissure syndrome+ CN V-
2(i.e maxillary division)

17. Answer is D.
Elliptical space between vocal cords(Thyroaretenoid
involved)
Triangular space in posterior glottis(Inter-aretenoid
involved)
Key hole appearance of glottis(both muscle involved)

18. Answer is B.
Stapedial reflex-
Afferent- VIII N.
Efferent- VII N.

19. Answer D
Since antibiotic treatment has been effective to relieve
pain and fever, the child is left with a sterile fluid in the
middle ear. If Ibis fluid does not drain spontaneously in
another 12 weeks treat the case as one of otitis media with
effusion.

20. Answer is C.
Choanal atresia is d/t persisting bucconasal membrane
McGovern's technique: Take an artificial nipple> pierce
the nipple> baby breaths through nipple by sucking

21. Answer is B.
Inspiration- Middle meatus
Expiration- Middle meatus(mostly), inferior

22. Answer C
In long-standing middle ear effusion, fibrous layer of
tympanic membrane undergoes dissolution and as a result
becomes thin and atrophic and easily amenable 10 form
retraction pockets or cholesteatoma. Stasis of secretion
also causes cholesterol granuloma in the mastoid.
Ossicular necrosis rather than fixation is commonly seen.
Long process of incus and stapes superstruclore undergo
necrosis.

23. Answer C )
Mastoid swelling, fever and pulsatile ear discharge that the
child has developed point to acute mastoiditis. Treatment
would include IN antibiotics, analgesics and cortical
mastoidectomy, Modified radical mastoidectomy is indicated
in cholesteatoma. Antihistamines have no role.

24. Answer is A.
Children- Ethmoidal
Adults- Maxillary
Polyps-opposite of sinusitis
Children-Maxillary
Adults-Ethmoids
Carcinoma-Maxillary
Mucocoele-Frontal-->X-Ray-->loss of scallops

25. Answer is A.
Nerve supply of Parotid-
Sensory- Auriculotemporal
Secretomotor- Glossopharyngeal

26. Answer B
Patulous eustachian tube is an abnormally patent tube seen in
the third trimester of pregnancy or in rapid weight loss.
Symptoms complained by her are due to autophony. Conductive
hearing loss due to otosclerosis, otitis media with effusion and
retracted tympanic membrane also conduct body sounds to the
ear but her tympanic membrane findings of movement with
respiration are typically seen in patulous eustachian tube.

27. Answer is B.
Optic-->Superolateral
Internal carotid-->Lateral

28. Answer A
Glomustumour is five times more common in women. It is a
very slow-growing tumour. Sometimes glomustumour does not
occur alone but is associated with chemureceptortumours, seen
in carotid body and on opposite side of the body signifying its
multicentric origin. These tumours are extremely vascular and
bleed profusely; diagnostic biopsy is therefore contraindicated.
Diagnosis is made by high resolution gadolinium enhanced CT.

29. Answer is B.
2-->formed due to sup. Semicircular canal
3--> thin plate of bone separating cranial and tympanic
cavities

30. Answer is A.
to rule-out nasopharyngeal carcinoma

31. Answer is C.
Never changed-->Usually kept < 48 hours
(when> 48 hrs-->give antibiotic coverage)

32. Answer D
Griesinger's sign is oedema over the mastoid due to thrombosis
of mastoid emissary vein. It is seen in lateral sinus thrombosis.
Other three are features of glomustumour when it arises from
dome of jugular bulb (glomusjugulare). It produces red flush in
the lower pan oftympanic membrane resembling a "rising sun",
Brown's signis elicited with pneumatic otoscope when pressure
is raised in the ear canal,tumour pulsates vigorously and when
pressure is further raised pulsations stop altogether and
tumour blanches. Reverse occurs on decreasing the pressure. It
is also called "pulsation Sign".
Aquino's sign is blanching of tumour on compression of
ipsilateral carotid.

33. Answer is B.
Preauricular sinus-->If arch fusion is not complete
Callover fistula-->it is the 1st branchial cleft anomaly
Second bronchial fistula-
Sternocleidomastoid(Upper 1/3)-->internal opening-->
near tonsils
In close proximity to 2nd bronchial fistula--> Carotid
A.(int./ext.), hypoglossal nerve


34. Answer is D.
External ear is supplied by-
Auriculotemporal nerve(V3)
Greater auricular nerve(C-2,3)
CN VII & X
Lesser occipital nerve (C-2)

35. Answer is C.
Bony landmark(Anatomical/Surgical)-->McEvan's
triangle(Suprameatal triangle)

36. Answer is B.
Tragal sign--> Furunculosis

37. Answer is B.
When does the canal completes it's development is the
question
Clinical Importance
BAHA- Bone Anchoring Hearing Aid given upto 5-yrs
Ear surgery for atresia-->5-7 yrs-->when canal
development is full

38. Answer A
For a smalltumour confined to promontory, surgical excision is
the treatment of choice.
Embolisation reduces vascularity of the tumour and decreases
its size. It is used either pre-operatively before surgery or is the
sole treatment in inoperable patients who have received
radiation.
Radiation therapy reduces the vascularity of tumour and
arrests its growth. It is used in elderly patients or
inoperable large tumours, residual tumoursortumours that
have recurred after surgery.

39. Answer is B. (Adult-->12-15 mm)
40. Answer is C. (1.25 cm below and behind)
41. Answer is A. (base-->high frequency)

42. Answer D
Paraganglioma (glomustumour) arises from the
paraganglionic tissue normally present in the adventitia of
dome of jugular bulb, glomus body on the promontory of
middle ear, at the carotid bifurcation and glomus body
along the vagus nerve at the base of skull. They have been
called glomustympanicum, glomusjugulare, carotid body
tumourandglomusintravagale, respectively. Most common
tumours of facial nerve are neuroma and haemangioma.

43. Answer is B. (it is for middle ear Sx)
Special finding in petrositis-->Pulsatile discharge(also in
ASOM)
Pulsatile tinnitis-->Glomus tumor
Fluctuating tinnitis-->Meniere's disease

44. Answer is B.
M/C-->Posterosuperior quadrant

45. Answer is C
3-->when it progresses
Meneier's-->false positive

46. Answer is B. (blanching of tumor on siegel's
speculum)

47. Answer is D. (either gets blocked or dead)

48.Answer is A.
Canasopharynx is most commonly seen in fossa of
Rosenmuller
This fossa is just behind the Eustachian tube and hence a
Ca of this fossa often blocks the tube leading to Glue ear
A patient of glue ear will present with dull TM, deafness
and tinnitus, type B curve impedence. The most common
presentation of Canasopharynx is neck mass.

49. Answer is B.
(CROS-->Contralateral Routing of signals)
Normally both ears are needed for spatial orientation

50. Answer is C.
Post. Auricular abscess- commonest, abscess forms over
mastoid, pinna is displaced forwards.
Bezold's abscess- abscess deep to sternomastoid
Cittelli abscess- Pus breaks through inner table of
mastoid tip and travels along the posterior belly of
diagastric muscle. Swelling is seen in the diagastric
triangle of neck.

51. Answer is B.
Superior or post. superior (P=79)
Otoscopy finding-->hanging superior canal wall

52. Answer is C.
I/II Radiotherapy
III/IV Sx and radiotherapy
Keratinization-->Stripping

53. Answer is C.
At the junction of-->anterior 1/3 and posterior
2/3
Early-->Pinkish pale, mobile mucosa-->cured by
voice rest
Late-->White or gray, fixed mucosa--> voice rest--
>no improvement-->Sx

54. Answer is C.
Aspregillusfumigatus-->non-invasive-->Forms fungal
ball(Aspergilloma)
Mucormycosis-->dangerous-->since it is angioinvasive

55. Answer D
CSOM is notorious {or intracranial complications. The first
priority in mastoid surgery is given to free drainage of pus
to render the ear safe. Second priority is given to preserve
or reconstruct hearing mechanism.


56. Answer is B.
COWS (Dhingra-42)
Hyperactive-->Warm water like action
hypoactive-->Cold water like action

57. Answer is B.
Acoustic neuroma--> MRI--> Gold standard (P=110)
Angiofibroma Clinical Diagnosis (Dhingra-231)
Nasopharyngeal Ca MRI+Gadolinium (Dhingra-235)


58. Answer A
Otosclerosis mostly involves females, has a positive family
history and causes slow progressive bilateral hearing loss.
Audiogram is characterised by Carhart'snoteh-a dip in
booe conduction at 2000 Hz.

59. Answer B
Caucasians have highest prevalence. Japanese and Chinese
have low incidence. Lowest incidence occurs in African
Negroes.

60. Answer is B
Accessory maxillary sinus ostium is located in the lateral
wall of the middle meatus, usually in the anterior or
posterior fontanelle where the medial wall of the maxillary
sinus is membranous.

61. Answer is D
(may go in brain-->then segmental resection of brain)

62. Answer is B
M/C indication of tracheostomy: Ca larynx
Only indication for high tracheostomy: Ca larynx
M/C source of bleeding in tracheostomy- Inf. Thyroid
vein
M/C complication of tracheostomy-
Hemorrhage
in children-->Difficult decannulation
in adults--> Stenosis
Gold standard Rx of stenosis--> incision + end to end
stenosis
Structures damaged during tracheostomy-
Isthmus
Inf. Thyroid vein
Thyroidea ima artery
Only advantage of metallic tube in tracheostomy--> is in
cleaning and changing
How to clean non-metallic tube--> Instill Sod. Bicarb.-->
then suck
Shiley's/Koenig's tube>Paediatric
Montogmery--> T-Tube(Silicon)

Cricothyroidotomy and percutaneous tracheal ventilation
are preferred over tracheostomy in most emergency
situations. Disadvantage in tracheostomy is the inability to
place a tube of greater than 6 mm in diameter due to the
limited aperture of the cricothyroid space. It is also
relatively contraindicated in patients under 12 yrs of age
because of risk of damage to cricoid cartilage and
subsequent risk of subglottic stenosis.

63. Answer A
Otosclerosis is autosomal dominant disease.

64. Answer is A.
Reduce size and + fibrosis
But RT not used routinely-->since boy is juveline and
radiation increases bone Cancer
Normal Rx of angiofibroma-->Sx

65. Answer is B.
Parts of temporal bone- (MTP-SS)
Mastoid
Tympanic
Petrous
Squamous
Styloid process

66. Answer is 3.
Otosclerosis is a d/e of bony labyrinth(otic capsule)

67. Answer A
Most common site forotosclerosis to start is fissulaante-
fenestraman area lying in front of oval window. Other sites
listed in the question are also affected but their
involvement is less frequent

68. Answer is D.
More than 100 indications-->so go for all(always except
like obvious-appendectomy)

69. Answer is C.
SEE STEROIDS ARE USUALLY C/I IN FUNGAL
INFECTIONS
M/C cause of allergic sinusitis--> House dust

70.Answer is D.
Cochlear implants are the electronic devices which convert
the sound signal into the electric impulses that directy
stimulate cochlear nerve



71. Answer is D.
Ask EEE (adduction) and not AAA(Abduction)--> then
anterior commisure is seen (usually difficult to see in
indirect laryngoscopy)

72. Answer is B. (terbinate=concha)

73. Answer is A.
Sciebe's is the commonest aplasia
Congenital deafness--> U-Shaped audiogram

74. Answer C
Otic capsule (also called bony labyrinth) has three layers:
outer periosteal, inner endosteal and the middle bony. The
bony layer which develops from cartilage and for some
unknown reason bone of the otic capsule is absorbed and
replaced by spongy vascular bone which later becomes
sclerotic due 10 deposition of calcium.

75. Answer C
Tympanometry in stapedialotosclerosis shows AS curve. AD
curve is seen in ossicular discontinuity. Slowly progressive
conductive hearing loss in an adult with normal tympanic
membrane in appearance and mobility, and normal
function of eustachian tube, is typical of otosclerosis.
A Normal
As Reduced compliance at ambient pressure
(otosclerosis)
AD Increased compliance at ambient pressure
(ossiculardiscontinuity)
B Flat or dome shaped (fluid in middle ear)
C Maximum compliance at pressures more than
-100 mm H2O (negative pressure in middle ear)

76. Answer is C.
Type I- Myringoplasty
Type II- No Name
Type III- Columella
Type IV- No Name
Type V- Fenestration operation(fistula in horizontal SCC--
>vertigo)

77. Answer is B. (P-100 is not used now)

78. Answer D
Schwartze's sign is reddish hue seen over the promontory.
It is due to increased vascularity of bone and is seen in
early or active otospongiosis. Reddish hue seen through
intact tympanic membrane in its lower part is due to
glomusjugularetumour and is sometimes also referred to
as "rising-sun appearance". Better hearing ability in noisy
surroundings is a feature of otospongiosis and is called
paracusisWillisiana or paracusis of Willisi.

79. Answer is A.
Lymphoepithelioma-
M/C Site- nasopharynx
in fact most common type of nasopharyngeal Ca
(in WHO classification)
Rx- RT
Woodworkers- Adeno-->highly radioresistant-->Rx-->Sx

80. Answer is B.
chronic persistent positional vertigo--> singular nerve
neurectomy


81. Answer is A.
Active stage-->NaF-->then Sx

82. Answer is D. (P=340)

83. Answer is A.
HOLMAN MILLER SIGN (ANTRAL SIGN)- The anterior
bowing of the posterior wall of the antrum seen on lateral
skull film. Pathognomic for juvenile nasopharyngeal
angiofibroma

84. Answer B
A negative Rinne's test for 256, 512and 1024 Hz will show
a minimum AB gap of 15, 30 and 45 dB, respectively.

85. Answer D
Though A, B and C, are features of Van der Hoeve
syndrome, preauricular sinuses are not associated with
this syndrome.

86. Answer B
Carhart's notch is loss of bone conduction in audiogram. It
is a feature of otosclerosis. Loss s is
5dB in 500Hz
10 dB in 1000 Hz
15 dB in 2000 Hz
5 dB in 4000 Hz

87. Answer D
Carhart's notch is a sensorineural hearing loss centred
maximally at 2 kHz in bone conduction curve of the
audiogram. It is only an apparent loss due to stapes
fixation and can be reversed by stapedectomy.

88. Answer B
Stapedectomyor stapedotomywith a prosthetic connection
between incus and oval window is the treatment of choice.
Hearing aid can also be useful if patient refuses surgery.
Stapes mobilisation will also restore hearing but the
benefit is short-lived due to refixation. Fenestration
operation in which a fenestra is made in the horizontal
canal and covered with a tympanomeatal flap has been
abandoned as this leaves a mastoid cavity which requires
lifelong aftercare and also a hearing loss of 25 dB.
After stapedectomy there is potential risk of giddiness,
therefore it is not advised to high construction workers
and divers. It is also contraindicated in professions in
which patient has to strain or frequent pressure changes
occur in the middle ear. e.g. professional athletes or
frequent air travellers. In them there are chances of
developing perilymph fistula with consequent vertigo and
sensorineural hearing loss. Since the person in question is
an office-goer, it does not form a contraindication to
stapedectomy.

89. Answer is 2.
Incus (lenticular part most common)
M/C Site: IS joint(towards incus side)
Galle's test: -ve condition-->Otosclerosis, Ossicular Chain
disruption

90. Answer C
Tullio phenomenon is loud sounds or noise producing
giddiness. It is seen in congenital syphilis, Meniere's
disease and when three functioning windows are present
in the ear, i.e, round window, oval window and a third
artificially created window such as fenestration of the
lateral canal (an old operation for otosclerosis) or a fistula
on semicircular canal.
In Meniere's disease a distended saccule lies against stapes
footplate and in congenital syphilis adhesions form
between membranous labyrinth and the footplate. Thus
movement of stapes due to loud sounds stimulates
labyrinth in both these conditions.
ParacusisWillisiis seen in otoscleroties. They hear better in
noisy than in quiet surroundings.
Hennebert'ssign.It is a positive fistula test without the
presence of a fistula. It is seen in congenital syphilis or
Meniere's disease (25% cases).
Otolithic crisis of Tumarkin or drop attacks are seen in
early or late Meniere's disease, Patient feels as if pushed to
the ground without any vertigo or loss of consciousness. It
is presumed they are due to distortion of otolithic
membrane of utricle or saccule when endolymphatic
pressure rises.

91. Answer D
In the above question, patient is a female with bilateral
conductive hearing loss with normal tympanic membrane.
Also the As curve indicates that compliance of tympanic
membrane is restricted indicating ossicular fixation.
Absence of stapedial reflex indicates that contraction of
stapedius muscle is not producing any change in
compliance of 1M again indicating stapediallixation. The
diagnosis points to otosclerosis. Treatment of otosclerosis
includes A, B and C. Gentamicin is never used. Gentamicin
perfusion of the middle ear is used in Meniere's disease to
selectively destroy vestibular end organs and to stop giddy
attacks.


92. Answer is A.
Rt ear-->Rt hand; Lt ear-->Lt hand
Indirect laryngoscopy-->Left hand used (always)

93. Answer is B. (occurs d/t smoking)

94. Answer B
All of the above conditions cause conductive deafness.
Otosclerosis usually starts between 20 and 30 years. It is
rare before 10 years. Acute otitis media is usually
unilatera1 and presents with severe pain in the ear.
Congenital cholesteatoma is rare condition and extremely
rare to be bilateral. OME is the most common cause
affecting children.

95. Answer D
Acoustic neuroma and endolymphichydrops (Meniere's
syndrome) present with tinnitus, dizziness and
progressive hearing loss. This also holds true for
meningiomas which involve cerebellopontine (CP) angle.
Nearly 10% of tumours of CP angle are meningiomas.
Histiocytosis X. presenlly called Langerhans cell
histiocytosis, is a rare disease which may involve temporal
bone (as eosinophilic granuloma) or multiorgan disorder
involving skull, long bones, ribs. Vertebrae, pelvis, maxilla
and mandible, and other non-osseous organs. Langerhans
cells are involved in cell-mediated immunity, cause
osteolytic lesions in bones and recruit eosinophils.
Bosioophilic granuloma is a milder form of Langerbans cell
histiocytosis which can involve temporal bone.

96. Answer is C.
Tonsil--> deep lobe involved--> Total parotidectomy
Usually superficial lobe is involved--> Superficial
dissection

97. Answer is B. (least imp, also benign)

98. Answer is B. (most commonly injured)
Parotid gland-->Stenson's duct
Submandibular gland--> Wharton's duct

99. Answer C
The patient suffers from sensorineural deafness, so fenestration
and stapes mobilisation cannot be done. Such patients require
hearing aid or cochlear implant. Since hearing aid has not shown
any benefit, cochlear implant will be indicated to provide
hearing and develop speech and language, There is no
conservative treatment.

100. Answer is C.
Medial geniculate body of thalamus-->Perception possible
Frequency coding-
Cochlea-
Apical-
Basal- low frequency(ck)

101. Answer D
Tympanic membrane develops from all the three germinal
layers. Outer epithelial layer develops from epithelium lining the
first branchial cleft
Inner mucosal layer develops from the lining of tubotympanic
recess which is a derivative of first pharyngeal pouch and partly
of the second pouch.
Intermediate fibrous layer is derived from mesoderm.

102. Answer D
Bony labyrinth, also called the otic capsule, develops from
cartilage which later ossifies to form bone. There are 14 centres
of ossification. Ossification starts at 16th week when first centre
appears near the cochlea. The last centre appears at 20th week
at the posterolateral part of the posterior semicircular canal.

103. Answer C )
Lateral surface of tympanic membrane is supplied by
auriculotemporal nerve in its anterior half and auricular branch
of vagus in the posterior half. The medial surface in its entirety is
supplied by CN IX through its tympanic branch.

104. Answer B
Round windowis covered by secondary tympanic membrane.
Stapedius muscle arises within the hollow body of pyramid and
stapedial tendon comes out of its tip to get attached to neck of
stapes.
Sinus tympani is an open deep depression in the medial wall of
middle ear. It lies medial to vertical part offacial nerve and the
pyramid. Above it is bounded by ponliculus-a bony bar
connecting pyramid to promontory-and below by the subiculum.

105. Answer B
Though pinna has attained the adult configuration (shape) by
birth, it continues to grow in size after birth and attains 90-95%
of the adult size by the age 5-6 years. This is the age when
plastic surgical correction of pinna can be done.

106. Answer C
Glossopharyngeal nerve (CN IX) gives a branch called
Jacobson's nerve (tympanic branch of glossopharyngeal)
which along with caroticotympanic nerves fonns tympanic
plexus on the promontory of middle ear. Glossopharyngeal
nerve is sensory while caroticotympanic nerves carry
sympathetic fibres. Tympanic plexus supplies innervation
to the medial surface of tympanic membrane. tympanic
cavity, mastoid air cells and bony eustachian tube.

107. Answer C
Sinodural angle is also called the Citelli's angle. It is formed
at the site where dura of middle cranial fossa meets the
dura of sigmoid sinus. It forms an important landmark in
mastoid surgery.
McEwen's triangle is bounded by temporal line,
posterosuperior segment of the external auditory canal
and a line drawn as a tangent 10 the external canal. It is an
important landmark to open the mastoid anl111lll-an
initial step for mastoidectomy.
Solid angle is the place where three semicircular canals
meer.

108. Answer A )
Total length of external auditory canal from concha to
tympanic membrane is 24 mm (compare eustachian tube
which is 36 mm), Its outer one-third is cartilaginous while
inner two-third are bony. Hair follicles, sebaceous and
cerumInous glands are confined only to its outer one-third.
Boil. a staphylococcal infection of the hair follicle,
therefore occurs only in the outer part of the canal.
Floor of the cartilage forming outer cartilaginous canal
may show dehiscences called fissures of Santorini, which
permit infections of the parotid or from the mastoid to
present in the canal and vice versa.

109. Answer A
Komer's septum is a bony plate, sometimes present in the
mastoid, dividing superficial squamous cells from the
deeper petrosal cells. Antrum lies deep to it Antrum cannot
be located unless this septum is removed during mastoid
exploration, Facial recess lies lateral to sinus tympani. Middle
ear can be entered through the facial recess in combined-
approach tympanoplasty or surgery for cochlear implant.

Trautmanns triangle is a plate of bone between solid.
Angle, sinodural angle and sigmoid sinus (Fig. 1.2). It is a
landmark for entry into the posterior crania1 fossa.
Arcuate eminence is seen in superior surface of petrous
bone. Underneath this eminence lies the superior
semicircular canal.

110. Answer D
Normal tympanic membrane shows mobility on siegalisation.
These movements are lost or, restricted in middle ear effusion or
when tympanic membrane is thick or markedly retracted.
The tympanic membrane becomes congested as a result of crying,
sneezing orblowingofnose.Itis common to see a congested
tympanic membrane in a crying child.
In a bulging tympanic membrane lateral process of malleus
audits handle maybe obscured.

111. Answer C
The nerve trunk lies 6-8 mm deep to tympanomastoidsuture, It
is the most reliable landmark

112. Answer C
Endolymph is secreted by striavascularis which forms the outer
wall of scala media (also ca1led the cochlear duct). It is absorbed
through endolymphatic sac.

113. Answer C
In radical mastoidectomy, middle ear is exteriorised into extemal
ear cana1 and the eustachian tube is blocked by packing it with
muscle or cartilage to prevent nasopharyngeal infections
reaching the middle ear.

114. Answer B

Stapes superstructure, i.e. its head, neck And both crura develop
from Reicben'seartilage (second arch).
Stapes footplate has a dual origin both from otic capsule and
second arch.

115. Answer A
Due to changes in maturation of central nervous system, findings
of BERA change during infancy. Therefore, only age specific
latency and intensity findings are used to interpret results.
However, it is not affected by sleep, sedation or attention of the
child. In fact sedation is used to elicit responses in infants and
children.

116. Answer E
Radiation, rubella (German measles), thalidomide
administration or infection with cytomegalovirus during
pregnancy have a teratogenic effect on the development of
inner ear particularly in the first trimester,

117. Answer C
Studies in experimental animals and human temporal bones
have shown oedema of striavascularis and reduced blood
supply to lateral wall of cochlea, This action on striavascularis
results in reduction in endocochlear potentials and thus cause
rise in threshold of compound action potential. Furosemide
causes both temporary and permanent hearing loss.

118. Answer A
Jacobson's nerve is the other name for tympanic branch of
glossopharyngeal nerve (CN IX). It supplies sensory innervation
to mucosa of the middle ear and mastoid and also carries
preganglionic parasympathetic secretomotorfibres to the
parotid gland via lite lesser petrosal nerve and otic ganglion.
Section of these fibres interrupts secretomotor nerve supply to
the parotid and thus helps in the relief of gustatory sweating
(Frey's syndrome).

119. Answer C
Normally in a sitting position, lateral canal is tilted 300
backwards. By extending the bead 60 backward, canal assumes
a vertical position which is ideal for caloric test.

120. Answer B
Dorello'scannltransmits CN VI. It lies between petrous apex
laterally and pelroclinoid ligament superornedially. It is
involved in petroSitistheGnldenigo's sign.

121. Answer D
Costen's syndrome is due to temporomandibular (TM) joint
abnormality with defective bite. It causes pain in the ear and the
surrounding frontal, parietal and occipital regions, along with
vertigo and tinnitus. It is treated by analgesics, local heat and
slow exercises of TM joint. Defective bite can be corrected by an
orthodontist.

122. Answer A
Coalescent mastoiditis is treated by cortical
mastoidectomy, also known as Schwartz operation. In this
operation all mastoid air cells are exenterated leaving
posterior canal wall intact.

123. Answer C
CN X (vagus) nerve supplies both the ear and respiratory
tract and lungs. Manipulation of ear canal such as removal
of wax from the canal provokes cough.

124. Answer B
Negative Rinne on the right means bone conduction better
than air conduction, meaning thereby conductive deafness.
But this test has been done without masking the left ear
which is normal in this case. Response to bone conduction
could be from the left ear. Thus right ear suffers from
sensorineural deafness. Rinne's test in this case is "false
negative".

125. Answer B
Character of tinnitus in Meniere's disease is roaring or
hissing type. Pulsatile tinnitus (non-continuous tinnitus) is
due to vascular or nonvascular causes. Common
aetiologies include:
(i) Raised intracranial pressure including benign
intracranial hypertension
(ii) Glomustumours
(iii) Hypertension
(iv) Venous hum
(v) High jugular bulb
(vi) Atherosclerotic carotid artery disease
(vii) Arteriovenous fistula or malformation
(viii) Thyrotoxicosis, pregnancy due to increased cardiac
output (ix) PersIstentstapedial artery (rare)
(x) Palatal myoclonus or myoclonus of tensor tympani or
stapedius muscle (non-vascular).

126. Answer C
Malignant otitis extema is not a neoplastic disease. The
term malignant is used because of lhe severity of disease
which can cause several complications including multiple
cranial nerve palsies. Malignant otitis externa is an
infective process caused mostly by P. aeruginosa. II affects
elderly diabetics or Ibose with immunocompromised
status.

127. Answer C
Treatment of glue ear where thick secretions accumulate
in middle ear would be drainage through a myringotomy
and long-term aeration of middle ear through a grommet.
Options B and D are only myringotomy which will soon
close leading to recurrence of disease.

128. Answer B
Facial nerve palsy occurs in 50% of transverse and less
than 20% of longitudinal fractures. Palsy is not always due
to transection of nerve. It C&1 occur will the formation of
intraneuralhaematoma or a spicule of bone embedded iii
the nerve. CSF otorrhoea does not occur in all cases of
fractures.

129. Answer B
The option B should have been primary cholesteatoma and
not secondary.
Secondary cholesteatoma occurs due to migration of
squamous epithelium through a preexisting perforation
such as posterosuperior marginal or total perforation (as
occurs after acute necrotising otitis media). There is no
term like tertiary cholesteatoma as given in option D.

130. Answer C
Aim of surgery in middle ear infections is to make the ear
safe. Since attic cholesteatoma in Ibis case is complicated
by lateral sinus thrombosis, modified radical
mastoidectomy (canal wall down procedure) will be
justified. In this case, the disease will be exteriorised so
that the cavity could be subsequently examined and
cleaned if cholesteatoma was left behind or recurs.
Intact canaI wall mastoidectomy and simple
mastoidectomy are "canal wall up" procedures.
Obliteration of mastoid cavity even after removal of
cholesteatoma runs the risk of burying cholesteatoma and
subsequent complications.

131. Answer B
Impedance audiometry is not used in children before 4
months of age because of the characteristics of ear canal.
Ear canal is collapsible and a true tympanogram is difficult
to get. Similarly behavioural audiometry and free field
audiometry arc not possible in a neonate. They cannot test the
level of hearing. Results of behavioural audiometry will vary
due to subjective observation.
The only reliable method is brain-stem evoked response
audiometry (BERA). In this air conducted clicks arc presented
through headphones and response picked up from surface
electrodes applied to scalp. They arc also not affected by sleep
and sedation. Latency and morphology of BERA waves changes
rapidly with age in infancy and therefore only age specific
norms of latency and amplitude are used to interpret results.

132. Answer A
Note that ossicular disruption with intact tympanic membrane
(TM) causes more hearing loss than ossicular disruption with
perforated tympanic membrane.
Hearing loss caused by different lesions of conducting
apparatus is as follows:
(i) Complete obstruction of ear canal: 30 dB
(ii) Ossicular disruption with intact TM: 54 dB
(iii) Ossicular disruption with TM perforation: 38 dB
(iv)Perforation of TM: Varies from 10 to 40 dB depending on
size and site of perforation
(v)Otitis media with effusion (fluctuating hearing loss): 2040
dB
(vi) Complete closure of oval window: 60 dB

133. Answer C
A Siegel's speculum bas a magnifying glass and pump attached
to it.The pump helps 10 increase or decrease air pressure in the
ear canal while lens gives magnification. If a powder like
antibiotic or antiseptic (iodine plus boric acid) is placed in the
speculum, it can be blown in the ear for treatment of otitis
media or otitis externa. A Siegel's speculum is used in the
following conditions:
(i) To magnify tympanic membrane to see small perforations.
(ii') To see mobility of tympanic membrane.
(iii) To perform fistula lest.
(iv) To insufflate powder into the ear canal and middle ear.

134. Answer A
Gradenigo's syndrome classically consists of a triad of (i)
deep-seated or retro-orbital pain, (ii) diplopia due 10 CN
VI involvement,and (iii) otorrhoea due to middle ear or
mastoid infection. II is seen in petrositis where infection
from the middle ear and mastoid spreads to the petrous
apex with formation of extradural abscess. II may be
associated with several other symptoms such as earache,
facial paralysis, acoustic or vestibular symptoms, or
hearing loss, but they are not included in the syndrome.

135. Answer C
In Weber's test, tuning fork is placed on the forehead or
vertex in the midline. Sound is conducted equally 10 both
ears through bone vibration. In a normal person.the sound
is centralised as it is beard equally in both ears.
In conductive deafness, sound is localised 10 the affected
ear (in this case right ear) because cochlear function is
normal. The cochlear function is normal in healthy ear also
but the ambient noise of air-conducted sound produces a
masking effect on the normal side. Masking effect of air-
conducted sound on the diseased ear (right ear in this
case) is missing.

136. Answer B
Bilateral hearing loss getting worse in pregnancy is due 10
otosclerosis. Tympanogram in this case would be A. Iype. It
is a normal Iympanogram with reduced compliance due 10
stapes fixation.

137. Answer A
Pure tone and speech audiograms are measured in dB HL
(hearing level). In normal bearing adult person bearing
threshold is 0 dB HL. dB SL (sensation level) is the
intensity of sound that will produce same sensation of
hearing as in a normal person. Thus in a hearing impaired
individual who already has a hearing loss of 20 dB, a sound
of 50 dB will produce a sensation of 30 dB only. dB SPL
(sound pressure level) is the physical measure of sound
intensity. Environmental sounds (noise pollution levels) are
measured in dB A. dB SHL is used in auditory brain-stem
response to click stimulus.

138. Answer B
Otoacoustic emissions (OAEs) are produced by the outer hair
cells. Motility of outer hair cells, spontaneous or in response to
sounds, is transmitted by the basilar membrane, ossicular chain
to tympanic membrane, and can be picked up and measured by a
sensitive microphone placed in the outer ear canal.
Spontaneous OAEs are present in 70% of normal individuals.
Evoked OABs are produced in response of pure tones or clicks.
Absence of OABs indicates damage to outer hair cells as in
ototoxicity.

139. Answer C
500, 1000 and 2000 Hz are the speech frequencies. Average
ofthreshold levels at these frequencies is called pure lone average
(PTA). PTA should agree with 7 dB of speech reception
threshold (SRT). IfSRTisunusually better relative to PTA, it
indicates non-organic hearing loss or malingering. PTA is also
used to measure percentage of hearing impairment in one or
both ears and in measurement of the hearing handicap.

140. Answer is A

141. Answer is C. (Posterior part of the nasal cavity):
Nasopharyngeal Fibroma (Juvenile nasopharyngeal
angiofibroma) Origin -
i) Arise from the posterior part of the nasal cavity
close to the superior margin of sphenopalatine
foreman.
ii) Usually arise from the lateral wall of the
nasopharynx (III-LT)
Commonest benign tumour of the nasopharynx, young
boy (10-20 years age)
Profuse and recurrent episistaxis is the most common
presentation
Progressive nasal obstruction and de-nasal speech
Conductive hearing loss and serous otitis media
Mass in the nasopharynx - sessile, lobulated or
smooth and obstructs one or both choane pink or
purplish in colour.
Consistency is firm but digital palpation should never
be done until at the time of operation.
Other clinical features like broadening of nasal bridge.
proptosis. swelling of cheek, infratemporal fossa or
involvement of II, III, IV, VI, cranial nerve depend on
the extent of tumour
* Biopsy is avoided (contraindicated)
Investigation of choice - CT scan with enhancement
Surgical excision is now the treatment of choice.

142. Answer is A. (Middle meatus):
Openings in meatus
I. Superior meatus
II. Middle meatus




III. Inferior meatus
Sphene-ethmoidal
recess
Posterior ethmoidal sinus
Frontal air sinus
Maxillary air sinus
Middle (Anterior)
ethmoidal sinus
(Dacrocystorrhinostomy
openings)
Nasolacrimal duct
Sphenoidal air sinus

143. Answer is C. (Needle in trachea):
In emergency situation where facilities are not present
needle in the trachea is best option, otherwise,
tracheostomy should be done.

144. Answer is B. (Tracheostomy):
Indications for tracheostomy
A. Respiration obstruction
1. Infections
Acute laryngo-tracheo-brcnchitis, acute epiglottis,
diptheria
- Ludwig's angina, peritonsilar, retropharyngeal or
parapharyngeal abscess, tongue abscess.
2. Trauma
External injury of larynx and trachea
Trauma due to endoscopies, especially in infants
and children
Fractures of mandible or maxillofacial injuries
3. Neoplasms - Benign & malignant neoplasms of
larynx, pharynx, upper trachea, tongue and thyroid.
4. Foreign body larynx
5. Oedema larynx due to steam, irritant fumes or
gasses, allergy (angioneurotic or drug sensitivity),
radiation
6. Bilateral abductor paralysis
7. Congenital anomalies
- Laryngeal web, cysts, tracheo-oesophageal fistula
- Bilateral choanal atresia

145. Answer is B. (150 l):
Origin
Absorption
Perilymph
(i) From CSF
(ii) Direct blood filtrate
from the vessels of spiral
ligament
Endolymph
(i) Secreted by stria
vascularis or by the
adjacent tissue of outer
sulcus
(ii) Derived from
perilymph across
Reissner's membranes

Through aqueduct of
cochlea in subarachnoid
space

Saccusendolyrnphaticus


Stria vascularis


146. Answer is A. (Petro-squamous suture):
Mastoid develops from the squamous and petrous
bones. The petro squamosal suture may persist as a
bony plate the Korner's septum, separating
superficial squamous cells from the deep petrosal
cells.
Korner's septum is surgically important as it may cause
difficulty in locating the antrum and the deeper cells,
and thus lead to incomplete removal of disease at
mastoidectomy.
Mastoid antrum cannot be reached unless the Korner's
septum has been removed.

147. Answer is C. (1
st
branchial cleft):
External auditory meatus - develops from the first
branchial cleft by about the 16
th
embryonic week.
External ear canal is fully formed by the 28
th
week.
Auricle: First branchial cleft is the precursor of external
auditory canal. Around the axis.

148. Answer is C. (30 db):
TYMPANOPLASTY - reconstruction of sound conducting
mechanism of middle ear.
Type I


Type II


Type III



Type IV
Type IV
Defect is perforation of tympanic
membrane which is repaired with graft. It
is also called myringoplasty.
Defect is perforation of tympanic
membrane with erosion of malleus. Graft is
placed on the incus or remnant of malleus
Malleus and incus are absent. Graft is
placed directly on the stapes head is placed
directly on the stapes head. It is also called
myringostapedioxy or
colummellatympanoplasty Only the foot
plate of stapes is present
Stapes foot plate is fixed but round window
is functioning. In such cases another
window is created on horizontal
semicircular canal and covered with a graft.
Also called fenestration operation

149. Answer is A. (Tympanic annulus):
Landmark for posterior tympanoplasty
1
st
landmark - Facial canal
2
nd
landmark - Short process of incus
3
rd
landmark - Tympanic annulus
150. Answer is C. (Hypotympanum):
GLOMUS TUMOUR - most common benign neoplasm of
middle ear origin from the glomus bodies (consists of
paraganglionic cells derived from the neural crest)
Common in middle age (40-50 years). Female
preponderance.
(i) Glomus jugulare - They arise from the dome of
jugular bulb, invade the hypotympanum and jugular
foramen, causing neurological signs of IX to XII
cranial nerve involvements. They may compress
jugular vein or invade its lumen.
(ii) Glomus tympanicum - They arise from the
promontory of the middle ear and cause aural
symptoms with facial paralysis.

151. Answer is A. (Otosclerosis):
"Schwartz sign" - is a pink reflex, seen through intact
tympanic membrane in the area of oval window, It
indicates active otosclerosis usually during
pregnancy.
"Griesinger's sign" - is seen in lateral sinus
thrombosis. It is due to thrombosis of mastoid
emissary vein impending venous drainage and thus
causing oedema over the mastoid
"Thumb sign" - Acute epiglottitis
"Steeple sign" - Acute /aryngotracheobronchitis

152. Answer is C. (Conductive deafness):
Traumatic rupture of Tympanic membrane
a. Trauma due to hair pin, match stick or unskilled
attempts to remove a foreign body
b. Sudden change in air pressure ego a slap or kiss on the
ear or a sudden blast, Forceful valvalsa may rupture a
thin atrophic membrane
c.Pressure by a fluid column ego diving, water sports
or forceful syringing d. Fracture of temporal bone
Injuries of tympanic membrane may be associated with
facial paralysis or subluxation of stapes (vertigo and
nystagmus) and sensorineural hearing loss. In such
cases, urgent exploration may be required

153. Answer isA . (Surgery):
Tubotympanie or Safe type
Discharge

Perforation
Granulations
Polyp
Cholesteatom
a
Complication
Audiogram
Profuse,
mucoid,
odourless
Central
Uncommon
Pale
Absent
Rare
Mild to
Scanty,
purulent, foul-
smelling
Attic or
marginal
Common
Red and
fleshy
Present
moderate
conductive
deafness
Common
Conductive or
mixed deafness

Treatment
1. Tubotympanic type - Aim to control infection and
eliminate ear discharge and at a later stage to correct
the hearing loss by surgical means (Myringoplasty)
2. Attico-antral type - Surgical is the mainstay of
treatment commonly performed operations -
atticotomy, modified radical mastoidectomy and
rarely mastoidectomy.
154. Answer is D. (Fungal infection):
RHINOSPORODIOSIS - It is a fungal granuloma caused by
Rhinosporodiumseeberi
The disease mostly affects nose and nasopharynx
**The disease is acquired through contaminated water
of ponds and cow-pets
Presents as a leafy, polypoidal mass, pink to purple in
colour and attached to nasal septum or lateral wall,
sometimes it extends into the nasopharynx and may
hang behind the soft palate
The mass is very vascular and bleeds easily on touch
it surface is studded with white dots representing the
sporangia of fungus
Early stages complains of nasal discharge which is
often blood tinged or nasal stuffiness
Sometimes frank epistaxis is the only presenting
complaint
*Diagnosis is made by biopsy, not possible to culture
Treatment - Complete excision of the mass with
diathermy knife and cuaterisation of its base, Dapsone
has been tried with some success
Rhinoscleroma - caused by Klebsiella -- rhinoscleromatis
or French bacillus (Gsram negative bacillus)
Woody feel nose
Diagnosis by - Mikulicz cells and Russell bodies

155. Answer is A. (Surgical):
Differences between antrochoanal and ethmoidal
polyp
Antrochoanal polyp Ethmoidal
polyp
Age

Aetiology

Number
Laterality
Origin




Growth



Size & shape


Common in
children

Infection

Solitary
Unilateral
Maxillary sinus
near the ostium



Grows backwards
to the choana; may
hang down behind
the soft palate
Tri-lobed with
antral, nasal and
Common in
adults
Allergy or
multifactorial
Multiple
Bilateral
Ethmoidal
sinuses
uncinate
process, middle
turbinate and
middle meatus
Mostly grow
anteriorly and
may present at
the nares
Usually small
and grape like





Recurrence

Treatment
choanal parts,
choanal part may
protrude through
the choana fill the
nasopharynx
obstructing both
sides

Uncommon, if
removed
completely
Polypectomy,
endoscopic
removal or
Caldwell - Luc
operation if
recurrent
masses







Common


Endoscopic
surgery or
ethmoidectomy

156. Answer is A. (Anyrtioteslld):
1. Anterior wall- has a thin plate of bone which
seperates the cavity from internal carotid artery. It
also has two openings; the lower one from the
eustachian tube and the upper one for the canal of
tensor tympani muscle

157. Answer is A. (Stapedial paralysis):
Hyperacusis is the sensation of discomfort or pain on
exposure to loud noises due to stapedial paralysis or
loss of taste (Involvement of chorda tympani)
When the same tone is heard as notes of a different
pitch in either ear the condition is known as diplacusis
Bell's palsy - defined as idiopathic, peripheral
facial paralysis of acute onset, more common in
diabetic (angiopathy) and pregnant women
(retension of fluid)
Bell's phenomenon - up and out rolling of the eyeball
during forceful closure

158. Answer is A. (Sensorineural deafness not
improving with medical treatment):
Cochlear Implants - They are electronic devices which
converts sound signals into electrical impulses which then
directly stimulate the cochlear nerve. Thus they replace
the non-functional transducer system of hair cells of the
cochlea.
1. Cochlear implants are more useful in post-
lingually deaf patients i.e. those who lost their
hearing after acquisition of language
2. Congenitally deaf patients have not been benefited as
effectively
3. Useful in the totally deaf to give an auditory rhythm
signal which can greatly aid lip reading, and are to be
thought of as an adjunct to a rehabilitation
programme
4. Useful for patients who are unable to benefit from
conventional hearing aid.

159. Answer is C. (Surgical removal gives excellent
prognosis):
Ramsay-Hunt's syndrome I Herpes-Zoster oticus
Herpetic infection of the geniculate ganglion is often
associated with facial palsy, accompanying auditory
and vestibular symptoms.
Herpetic vesicles on the external ear and the inside the
cheek with a LMN facial palsy, can be associated with
sensorineural deafness and vertigo.
There may also be anaesthesia of face, giddiness and
hearing impairment due to involvement of V
th
andVIII
th

nerve.
Treatment is same as Bell's palsy
The prognosis is generally poorer than for a bell's palsy
160. Answer is A. (Loss of corneal reflex):
Cerebellopontine angle tumours
Most common tumour is acoustic neuroma (80%) they
arise from the Schwann or neurilemmal cells
Commonest nerve of origin is the superior
vestibular, followed by the inferior vestibular and
rarely the cochlear
Most common presenting symptoms are unilateral
deafness of tinnitus or combination of both, 70%
have some evidence of ataxia or balance testing
55% have trigeminal nerve impairment, the earliest
sign of which is loss of corneal reflex
A lesion at cerehellopontine angle is identified
by the presence of vestibular and auditory
defects and involvement of other cranial nerves
such as
161. Answer is B. (Laryngomalacia):
162. Answer is B. (Mumps):
Mumps tends to give a unilateral deafness rather than a
bilateral one.
Conductive hearing
loss
Sensorineural hearing
loss
1. Negative Rinne test
i.e. BC > AC
2. Weber lateralised
to poorer ear
3. Normal absolute
bone conduction
4. Low frequencies
affected more
5. Audiometry shows
bone conduction
better than air
conduction with
air-bone gap.
Greater the air -
bone gap, more is
the conductive loss
6. Loss is not more
than 60 dB
7. Speech
discrimination is
good
1. Positive Rinne test
i.e. air AC > BC
2. Weber lateralised to
better ear
3. Bone conduction
reduced on
schwaback and
absolute bone
conduction tests
4. More often
involving high
frequencies
5. No gap between air
and bone
conduction curve on
audiometry
6. Loss may exceed 60
dB
7. Speech
discrimination is
poor
8. There is difficulty in
hearing in the
presence of noise

163. Answer is C. (Subperiosteal abscess):
164. Answer is B. (Hypertrophy of the sebaceous
gland):
Rhinophyma or potato tumour is a slow growing benign
tumour due to hypertrophy of the sebaceous glands of the
tip of nose seen in case of long standing acne-rosacea
It presents as a pink, lobulated mass over the nose
with superficial vascular dilation mostly affects men
past middle age
Treatment - Paring down the bulk of tumour with
sharp knife or carbon dioxide laser and the area
allowed to re-epithelialise. Sometime tumour is
completely excised and the raw area skin-grafted.

165. Answer is B. (Tonsillectomy patients):
Hypertrophy of lingual tonsils -- Mostly it is a
compensatory hypertrophy of lymphoid tissue in
response to repeated infections in tonsillectomy
patients.
Usual complains are discomfort on swallowing
feeling of lump in the throat, dry cough and thick
voice Treatment - laser surgery

166. Answer is B. (Anterior ethmoid cells (Aggar-nasi):
The anterior group sinuses (frontal, anterior, ethmoidal
and maxillary) drains into the middle meatus via their
ostia (openings)
The posterior ethmoidal sinuses drains into the superior
meatus and the sphenoid sinus into the sphenoethmoidal
recess. At the anterior end of the middle meatus is a ridge -
the agger nasi. This represents the nasoturbinal found in
many mammals or the "fourth turbinate"

167. Answer is C. (5 -10 mm/min):
Mucociliary mechanism - It moves at a speed of 5 - 10
mm per minute and the complete sheet of mucous is
cleared into the pharynx every 10 - 20 minutes.
About 600 - 700 ml of nasal secretions are produced in
24 hours

168. Answer is A. (Deafness):
ASOM - The principal symptom is pain, sudden and
severe, which may waken the child screaming and crying
at night. There will usually be a history of a preceding
URTI, but this is not invariable. Unless the condition is
bilateral or unless there is defective hearing in the other
ear deafness may not be noted by the parents.
Hearing tests will show a conductive hearing loss of
moderate severity Rinne's test will be negative

169. Answer is C. (Anterior pharyngeal wall):
Hypopharynx (Laryngopharynx) is the lowest part of the
pharynx and lies behind and partly on the sides of the
larynx. Its superior limit is the plane passing from the
body of hyoid bone to the posterior pharyngeal wall, while
the inferior limit is lower border of cricoid cartilage,
where hypopharynx lies opposite the 3'd, 4
th
, 5
1
\ 6
th

cervical vertebrae
Clinically it is subdivided into three regions
1. Pyriform sinus
2. Post cricoid region
3. Posterior pharyngeal wall

170. Answer is B. (Post-cricoid region):
Plummer - Vinson syndrome (Peterson-Brown
Kelly Syndrome) consists of dysphagia,
hypochromic microcytic anaemia, angular
stomatitis glossitis and Koilonychia.
It is a common site for carcinoma in female suffering
from plummer-vinson syndrome is Post cricoid region
A curious radiographic finding of unknown cause is a
web at the postcricoid region

171. Answer is C. (Astimazolc):
Motion-sickness- characterised by nausea, vomiting,
pallor, and sweating during sea, air, bus or car travel in
certain susceptible individuals.
It can be induced by both real and apparent motion and
is thought to arise from the mismatch of information
reaching the vestibular nuclei and cerebellum from the
visual, labyrinthine and somatosensory systems. It can
be controlled by the usual labyrinthine sedative
Prokinetic drugs (Metoclopramide, Domperidone,
cisapride, Mosapride) promotes gastrointestinal
transit and speed gastric emptying.
In motion sickness - uses sedative antihistaminis, while
Astimazole is non sedative anti histaminics

172. Answer is D.(Posterior):
The positional vertigos (benign) - Characterized by brief,
sudden attacks of vertigo precipitated by head
movements. There are no other aural symptoms although
the vertigo may be accompanied by a slight felling of
nausea. The disease is thought to be due to inorganic
deposit on the cupula of the posterior semicircular canals
and there is often a preceeding virus type illness.
Similar symptoms can arise after a head injury
Central (Malignant) Positional nystagmus - seen with
tumours of the posterior cranial fossa or the mid-brain
with disseminated sclerosis and with vascular lesions,
vetigo is often less than expected

173. Answer is B. (15 - 20 - dB):
Rinne's test - In this test air conduction of the ear is
compared with its bone conduction.
i) Rinne test is cailed positive when AC is longer or louder
than BC it is seen in normal persons or those having
sensorineural deafness.
ii) A negative Rinne (BC> AC) is seen in conductive
deafness. A negative Rinne indicates a minimum air-bone
gap of 15-20 dB
A prediction of air-bone gap can be made if tunning forks
of 256, 512 and 1024 Hz are used
A Rinne test equal or negative for 256 Hz but positive
for 512 Hz indicates air bone gap of 20 - 30 dB
A Rinne test negative for 256 and 512 Hz but positive
for 1024 Hz indicates air-bone gap 000 - 45 dB
A Rinne negative for all the three tuning forks of 256,
512 and 1024 Hz indicates air-bone gap of 45 - 60 dB
Remember that a negative Rinne for 256, 512 and 1024 Hz
indicates a minimum AB gap of 15, 30,45 dB respectively

174. Answer is B. (1, 3 : 1):
175. Answer is B. (Grants test):
Tuning Fork Tests
a) Rinne test
b) Absolute bone conduction (ABC)
c) Schwabach's test
d) Weber test
e) Bing test
f) Gelle's test
Ability to understand speech and its relation to
speech discrimination score
SD score Ability to understand
speech
90 -100% Normal
76 - 88% Slight difficulty
66 -74% Moderate difficulty
40 - 58% Poor
<40% Very poor

176. Answer is D. (Cerebral hemisphere lesion):
Cerebellar hemisphere lesions
Marked ataxia of the trunk
Ipsilateral appendicular deficits (Ataxia, incoordination
and hypotonia of the limbs)
Past pointing nystagmus
Dysmetria dysarthria
Gaze-evoked nystagmus is the most common form of
jerk nystagmus
Vestibular nystagmus - results from dysfunction of
the labyrinth (Minieres disease) vestibular nerve
or vestibular nucleus in the brain stem.
Downbeat nystagmus occurs from lesions near the
craniocervical junction (chiari malformation, basilar
invagination) brainstem or cerebellar stroke lithium or
anticonvulsant intoxication, alcoholism and multiple
sclerosis
Upbeat nystagmus is associated with damage to
the pontinetegmentum from stroke,
demyelination or tumours (176-H)

177. Answer is A. (Diplopia):
Acute maxillary sinusitis - Most commonly it is viral
rhinitis, which spreads to involve the sinus mucosa. This is
followed by bacterial invasion.
Clinical features
1. Constitutional symptoms - fever, general malaise, and
bodyache.
2. Headache
3. Pain - over upper jaw or teeth.
4. Tenderness - Pressure or tapping over the anterior wall
of the antrum produce pain
5. Redness and oedema of cheek - commonly seen in
children. The lower eyelid may become puffy.
6. Nasal-discharge - Pus may be seen on the upper soft
palate on posterior rhinoscopy.

178. Answer is A. (Tissue invasion is seen):
Allergic fungal sinusitis - It is an allergic reaction
to the causative fungus and presents with sinu-
nasal polyposis and mucin. The latter contains
eosinophils, Charcot-Leyden crystals and fungal
hyphae.
There is no invasion of the sinus mucosa with fungus.
Usually more than one sinus are involved on one or
both sides.
** There may be expansion of the sinus or bone erosion
due to the pressure but no fungal invasion.
Treatment is endoscopic surgical clearance of the
sinuses with provision of drainage and ventilation.
There is combined with pre and postoperative systemic
steroids.

Fulminant fungal sinusitis - It is an acute presentation
and is mostly seen in immunocompromised or diabetic
individuals. Common fungal species are Mucor or
Aspergillus
A) Mucor causes rhinocerebral disease. Due to
invasion of the blood vessels, mucor fungus causes
ischaemic necrosis presenting as a black eschar.
B) Aspergillus - infection can also cause acute fulminant
sinusitis with tissue invasion. Such patients presents
with acute sinusitis and develop sepsis and other
sinus complications (242-D)

179. Answer is B. (Squamous cell carcinoma):
Nasopharyngeal carcinoma - Chinese have higher
genetic susceptibility. E. B. virus is closely associated. Air
pollution, smoking of tobacco, and opium nitrosamines
from dry salted fish, smoke from burning of incense and
wood have been incriminated.
Squamous cell carcinoma (85%) is most common
Commonest site of origin in fossa of Rosenmuller in the
lateral wall of nasopharynx.
Cervical lymphadenopathy - Most common (60-90%)
Trotter's triad:
1. Conductive deafness (eustachian tube blockage)
2. Ipsilateral temporoparietal neuralgia (Involvement of
cranial nerve V) and
3. Palatal paralysis (cranial nerve X)
Irradiation (6000-7000 rads) is the treatment of choice

180. Answer is B. (Glottic carcinoma):
Glottic carcinoma - Curative radiotherapy is reserved for
early lesions which neither impair cord mobility nor
invade cartilage or cervical nodes.
Cancer of the vocal cord without impairment of its
mobility gives a 90% cure rate after irradiation and
has the advantage of preservation of voice.
Radiotherapy does not give good results in lesions
with fixed cords, subglottic extension, cartilage
invasion, and nodal metastases. These lesions require
surgery.

181. Answer is B. (Modified radical mastoidectomy):
Modified radical mastoidectomy - Where as much as
the hearing mechanism as possible is preserved. The
disease process which is often localized to the attic and
antrum is removed and the whole area fully exteriorized
into the meatus by removal of the posterior meatal and
lateral attic wall.
Indication
1. Cholesteatoma confined to the attic and antrum
2. Localized chronic otitis media

182. Answer A
As the acoustic neuroma enlarges in cerebellopontine angle first
it causes pressure on the fibres of CN V affecting corneal reflex.
Motor fibres of CN V are quite resistant. Similarly motor fibres of
CN VII are also resistant, though sensory fibres may be affected
and cause diminished sensation in posterosuperior wall of
external auditory canal (Hitzelberger sign). CN IX and CN X are
affected in late stages.

183. Answer is A. (Parapharyngeal abscess):
Parapharyngeal abscess (Abscess ofpharyngomaxillary
or lateral pharyngeal space)
Infections can occure from - Acute and chronic
infections of tonsil and adenoid, bursting of
peritonsillar abscess, Dental infection usually comes
from the lower last molar tooth. Ear, Parotid,
retropharyngeal and submaxillary spaces, Trauma.
Clinical features
Anterior compartment
infections produces
Posterior compartment
involvement
TRIAD
i) Prolapse of tonsil and
tonsillar fossa
ii) Trismus (due to
spasm of medial
pterygoid muscle)
iii) External swelling
behind the angle of Jaw
There is marked
odynophagia
1. Bulge of pharynx
behind the posterior
pillar
2. Paralysis of IX, X, XI,
and XII and sympathetic
chain
3. Swelling of parotid
region.
There is minimal
trismus or tonsillar
prolapse
Acute retropharyngeal abscess - Commonly seen in
children below 3 years. It is the result of suppuration of
retropharyngeal lymph nodes secondary to infection in
the adenoids, nasopharynx, posterior nasal sinuses or
nasal cavity. In adult may results from penetrating injury.
Chronic retropharyngeal abscess results from
1) Caries of cervical spine
2) Tuberculous infection of retropharyngeallymph nodes
secondary to tuberculosis of deep cervical nodes.

184. Answer is C (Radiotherapy):
Treatment of stridor
Once the diagnosis has been made, treatment of exact
cause can be planned.
Oxygen, Tracheostomy, and sometimes steroids may
be useful.

185. Answer is A. (Interarytenoid fold):
"Pachydermia laryngis" - It is a form of chronic
hypertrophic laryngitis affecting posterior part of larynx
in the region of interarytenoid and posterior part of the
vocal cords
That hypertrophy of the interarytenoid area,
formerly known as pachydermia. It is frequently
due to acid from oesophageal reflux.
Clinically, patient presents with hoarseness or husky
voice, and irritation in the throat.
Sometimes showing ulceration due to constant
hammering of vocal processes as in talking forming -
called Contact ulcer.
The condition is bilateral and symmetrical. It does not
undergo malignant change.
Treatment - is removal of granulation tissue under
operating microscope.

186.Answer is D (Brain stem implant):
Brain stem implant would be the most appropriate
treatment for rehabilitation of a patient who has bilateral
profound deafness following surgery for bilateral acoustic
schwanoma.

Dhingra says "In unilateral acoustic neuroma, ABI is not
necessary as hearing is possible from the contralateral side
but in bilateral acoustic neuromas as NF2,
rehabilitation is required by ABI."

Number of questions are increasing in exams regarding
rehabilitation of hearing impaired, so I am providing a bit
detail of this topic.

Important topics related to Hearing Aids
CROS: (Contralateral routing of signals)
In this type, microphone is fitted on the side of the
deaf ear and the sound picked up is passed to the
receiver placed in the better ear.
Useful for the person withone ear severely
impaired & helps in sound localization comingfrom
the side ofdeaf ear
Cochlear Implants
Electronic devices whichconvert sound signal
toelectrical impulses which directly stimulate the
fibers of CN VIII.
Cochlear implantsreplace the non-functional
transducer system of hair cells ofcochlea.
Used incochlear type of sensorineural hearing loss
where nerve fibers remain intact and functional.
Electrode array consists of multiple electrodes
and isimplanted in the scala tympani of the cochlea.
It stimulates the fibers of CN VIII.

Selection criteria for adults: (post lingually
deafened adults are good candidates. Results are not
so good in pre-lingually deaf adults unless they had
been receiving aural-oral training for communication)
A. Age: 18 years or more
B. Bilateral severe to profound hearing loss
C. Limited or no benefit from hearing aids
D. No medical contraindication to undergo surgery

Selection criteria for children: (Auditory deprivation
in the early developmental period causes degenerative
changes in central auditory pathway. Now implants can
be used at an age as early age 12 months. Implants
can be used in both pre-lingually or post-lingually
deafened infantsands children. The criteria are:
A. Bilateral profound or severe to profound hearing
loss
B. Minimal or no benefit from hearing aid
C. No medical contraindication
D. Willingness and support of family to enroll the child
for post-implant training programme
Auditory Brainstem Implant (ABI)
This implant is designed tostimulate the cochlear
nuclear complex in the brainstem directly byplacing
the implant in the lateral recess of fourth ventricle.
Such implants areneeded when CN VIII has been
severed in surgery of
vestibular schwannoma. In these cases, cochlear
implants are of no use.
In unilateral acoustic neuroma, ABI is not necessary
as hearing is possible from trhe contralateral side but
in bilateral acoustic neuromas as NF2 rehabilitation
is required by ABI.

187.Answer is A (Absent corneal reflex):
Dhingra 4th/m True about CP angle tumour is Absent
corneal reflex.

Acoustic neuroma may present with reduced corneal
sensitivity as well as pain, tingling, numbness on face due
to involvement of 5th cranial nerve.

Acousticneuroma
Acoustic neuroma is a benign, encapsulated,
extremely slow growing tumor of the 8th nerve.
(Commonest nerve of origin of acoustic neuroma
is the Schwann cells of superior vestibular nerve,
followed by inferior vestibular and rarely
cochlear)
Presenting symptom is progressive unilateral
Sensorineural hearing loss. This is accompanied by
tinnitus.
Clinical features
1. Age group of 40-60 years.(M:F=1:1)
2. Cochleo-vestibular symptoms are the earliest
symptoms when tumor is still intracanalicular and
are caused by pressure on cochlear or vestibular
nerve fibers or on internal auditory artery.
Progressive unilateral senssorineural hearing
loss, often accompanied by tinnitus is the
presenting symptom in majority of cases.

Cochlear Vestibular
Progressive
unilateral
Sensorineural
hearing loss
Tinnitus
Difficulty in
understanding
speech
Imbalance
Unsteadiness
Vertigo

3. Cranial nerve involvement:
Vth
- Reduced
corneal
sensitivity
and
numbness
of face
Vllth
-
Hypoasthesia
of posterior
meatal wall
- Loss of
taste
- Decreased
lacrimation
IXth&Xth
-
Dysphagia/hoarseness

4. Brainstem involvement
5. Cerebellar involvement
6. Raised intracranial tension
Remember:
MRI with gadolinium contrast is the gold standard
for diagnosis of acoustic neuroma.
Acoustic neuroma can arise from any nerve except
Optic & Olfactory because they are myelinated by
Oligodendroglia rather than Schwann cells.

188. Answer is A (Single and unilateral):
Antro-choanal polyp are usually Single and unilateral.

Antrochoanl polyp Ethmoidal Polyp
Age: common in
children
Etiology: Infection
Number: Solitary
Laterality:
Unilateral
Origin: Maxillary
sinus, near the
ostium
Grows backward to
the choana, may hang
down behind the soft
palate
Size & shape:
Trilobed with antral,
nasal and choanal
parts. Choanal part
may protrude
through the choana &
fill the nasopharynx
obstructing both
sides
Recurrence:
uncommon
Treatment:
polypectomy;
endoscopic removal
or Caldwell-Luc
operation if recurrent
Common in adults
Allergy or
multifactorial
Multiple
Bilateral
Ethmoidal sinuses,
uncinate process,
middle turbinate and
middle meatus
Mostly grow
anteriorly and may
present at the noses
Usuallysmall and
grape like masses

Common
Polypectomy;
Endoscopic surgery or
ethmoidectomy
(which may be
intranasal, extranasal
or transantral)

189. Answer is D (Palatal branch of sphenopalatine):

Little's Area or Kisselbach's plexus
Vascular area in the antero-inferior part of
nasal septum just above the vestibule
Common site of epistaxis
Also thesite for origin of the bleeding
polypus (hemangioma) of nasal septum

Plexus is formed by anastomosis of following
vessels:
Anterior ethmoidal
Sphenopalatine
Greater palatine
Septal branch of superior labial arteries

190. Answer is B (Mass):
Nasopharyngeal carcinoma presents as Mass.
Cervical lymphadenopathy (mass) is MC presenting
symptom(60-90%) in
Nasopharyngeal carcinoma.

Nasopharyngeal carcinoma
Epidemiology & Geographic distribution:
- Nasopharyngeal cancer is most common in China
particularly in southern states and Taiwan.
- Burning of incense or wood (Polycyclic
hydrocarbon), use of preserved salted fish
(Nitosamines) along with vitamin C deficient diet
(vitamin C blocks nitrosification of amines and and is
thus protective) may be other factors operative in China.
- People in Southern China, Taiwan and Indonesia are
more prone to this cancer
Aetiology: Exact etiology is not known. The factors
responsible are:
- Genetic: Chinese have higher genetic susceptibility to
nasopharyngeal cancer.
- Viral: Epstein-Barr virus is closely associated with
nasopharyngeal cancer.
Environmental: Air pollution, smoking of tobacco and
opium, nitrosamines from dry salted fish, smoke from
burning of incense and wood have all been incriminated.

Pathology: Squamous cell carcinoma in various
grades of its differentiation or its variants as transitional
cell carcinoma and lymphoepithelioma, is the most
common.
- The commonest site of origin is Fossa of Rosenmuller
in the lateral wall of Nasopharynx.
Clinical features:
Age: It is mostly seen in fifth to seventh decades but
may involve younger age groups.
Sex: Males are three times more prone than females.
Sympatomatology: Cervical lymphadenopathy is MC
presenting symptom (60-90%).
1. Nasal: Nasal obstruction, nasal discharge, denasal
speech (rhinolaliaclausa) and epistaxis.
2. Otologic: Due to obstruction of Eustachian tube,
there is conductive hearing loss, serous or
suppurative otitis media.
Tinnitus and dizziness may occur. Presence of unilateral
serous otitis media in an adult should raise suspicion of
nasopharyngeal growth.
3. Ophthalmoneurologic: Involvement ofIXth, Xth and
Xlth cranial nerves may occur, constituting jugular
foramen syndrome. Nasopharyngeal cancer can cause
conductive deafness (Eustachian tube blockage),
ipsilateral temporoparietal neuralgia (involvement
of CN V) and palatal paralysis (CN X)-collectively called
Trotter's triad.
4. Cervical nodal metastases: This may be the only
manifestation of nasopharyngeal cancer. A lump of
nodes is found between the angle of and the mastoid and
some nodes along the spinal accessory in the posterior
triangle of neck. Nodal metastases are seen in 75% of
the patients, when first seen, about half of them with
bilateral nodes.
5. Distant metastases involve bone, lung, liver and
other sites.

Diagnosis:
- Skull X-rays, tomograms or preferably CT scans are
done to demonstrate erosion of bone at the base of skull
and the extent of tumor.
Biopsy is essential to show the exact histology of the
malignancy. In absence of nasopharyngeal lesion but
with strong suspicion of malignancy, nasopharynx is
exposed by transpalatal approach and a strip of
mucosa and submucosa from the region of fossa of
Rosenmuller should be taken and subjected to
histology.
Treatment: Irradiation is treatment of choice.

191. Answer is A (Streptococcus):
Strep throat: Medline plus, Medical encyclopedia White
patch in the throat may be due to Streptococcus.

Strep Throat
Strep throat is an infection caused by
group A Streptococcus bacteria
Symptoms
Difficulty swallowing, Fever that begins
suddenly
General discomfort, uneasiness or ill
feeling
Loss of appetite, Nausea, Rash
Red throat, sometimes with white
patches
Sore throat, Tender, swollen lymph nodes
in the neck

192. Answer is D (Loss of timbre of voice):
Injury to superior laryngeal nerve causes Loss of timbre of
voic
Paralysis of Superior Laryngeal Nerve
Unilateral Bilateral
Paralysis of
superior laryngeal
nerve
causesparalysis of
cricothyroid
muscle and
ipsilateral
anaesthesia of
larynx above vocal
cord.

Clinical Features:
Voice is weak and
pitch cannot be
raised
Anaesthesia of the
larynx on one side
may pass
unnoticed or
causes occasional
aspiration
Laryngeal
findings
includeaskew
position of glottis,
shortening of cord
with loss of
tension, flapping
of paralyzed cord
Both the cricothyroid
muscles are paralyzed
alongwithanaesthesia of
upper larynx
Etiology: Surgical or
accidental trauma, neuritis
(mostly diptheritic), pressure
by cervical nodes or
involvement in neoplastic
process

Clinical features:
Presence of bothparalysis
and lateral anaesthesia
causes inhalation of food and
pharyngeal secretions giving
rise to cough and choking fits.
Voice is weak and husky

Treatment:
Cases due toneuritis may
recover spontaneously
Tracheostomy for repeated
aspirations
Epiglottopexy (an reversible
operation to close the laryngeal
inlet to protect the lungs from
repeated aspirations)

193. Answer is A (Anterior 1/3 and posterior 2/3
junction):
Most common location of vocal nodule Anterior 1/3 and
posterior 2/3 junction.

Vocal Nodules (Singer's or Screamer's Nodes)
Appearsymmetrically on the free edges of vocal cord
At thejunction of anterior one third, with the
posterior two-third, as this is the area of maximum
vibration of the cord and subject to maximum trauma
Size varies from pin head to half a pea
Occur due to vocal trauma when person speaks in
unnatural low tones for prolonged periods or at high
intensities
They mostly affectteachers, actors, vendors or pop
singers
Pathology:
Trauma to the vocal cord in the form of vocal abuse or
misusecausesoedema and hemorrhage in the
submucosal space.
This undergoes hyalinization and fibrosis, overlying
epithelium also undergoes hyperplasia forming a
nodule
Clinical Features:
Hoarseness, vocal fatigue and pain in the neck on
prolonged phonation
Treatment
Early cases can be treated conservatively (proper use
of voice)
Surgery is required for large nodules or nodules of
long standing in adults,
excised with precision under operating microscope
Speech therapy ad re-education in voice
productionare essential to prevent their recurrence.

194. Answer is D (Gentamycin):
This is the case of otosclerosis and all constitute part of
treatment except Gentamycin.

Otosclerosis (Otospongiosis)
Disease of bony labyrinth where the normal dense
enchondral bone is replaced by foci of irregularly laid
spongy bone.
50% patients have positive family history, inherited
as autosomal dominant
White races are affected more than negroes
Female are more common than males
Age of onset is around puberty
Types of otosclerosis:
1. Stapedial Otosclerosis:
Stapedial otosclerosis causingstapes fixation and
conductive deafness is the most common variety.
Lesion starts just in front of the oval window in an area
called fissula ante fenestrum
2. Cochlear Otosclerosis: Involves region of round
window
3. Histologic Otosclerosis: remains asymptomatic
Clinical Presentation:
Painless & progressive, bilateral (often) conductive
hearing loss / deafness is presenting symptom
Paracusiswilsii: patient hears better in noisy than quiet
environment
Tinnitus, vertigo andmonotonous speech are other
features
Tympanic membrane is usually normal and mobile
Schwartz Sign: a reddish hue or Flamingo pink tinge
may be seen on the promontory through the tympanic
membrane, indicating a focus of increased
vascularity.
Carhartz notch: A dip in bone conduction may be
seen which is different at different frequencies
butmaximum at 2000 Hz.
Treatment:
There is no medical treatment that cures otosclerosis,
sodium fluoride has been used to attain maturity of
active focus.
Stapedectomy with prosthesis replacement is the
treatment of choice
Hearing aid for patients who refuses or unfit for
surgery

195. Answer is B (Meningitis): Commonest complication of
CSOM is Meningitis.

Meningitis
MC intracranial complication of otitis media
Can occurin both acute and chronic otitis media
In adults, it follows chronic ear disease, which spreads
by bone erosion or retrograde thrombophlebitis
Clinical features: high grade fever, raised
ICT,headache, neck rigidity,projectile vomiting
Treatment:
Medical treatment takes precedence over surgery
Meningitis followingacute otitis media may require
myringotomy or cortical mastoidectomy.
Meningitis following chronic otitis media with
cholesteatoma require radical or modified radical
mastoidectomy

196.Answer is B (Pyogenic meningitis):
Patient presents with high fever, signs of raised ICT and a
past history of chronic otitis media likely diagnosis is
Pyogenic meningitis.

197.Answer is A (Oval window):
The part most commonly involved in otosclerosis is Oval
window.

198.Answer is D (Carcinoma larynx):
The most common cause of laryngeal stridor in a 60 years
old Carcinoma larynx.
Glottic cancer is MC site of CA Larynx. Hoarseness is MC
and the earliest symptom because of this glottic cancer is
detected early. Disease is mostly seen in
the age group of 40-70 years.

Cancer Larynx
It is 10 times more common in males than females.
Disease is mostly seen in the age group of 40-70
years.
Both tobacco and alcohol are well established risk
factor in laryngeal cancer.
Cigarette smoke contains benzopyrene and other
hydrocarbons which are carcinogenic in man.
About 90-95% of laryngeal malignancies are
squamous cell carcinoma with various grades of
differentiation.
Cordal lesions are often well-differentiated while
supraglottic ones are Anaplastic.
1. Supraglottic Cancer:
Supraglottic cancer is less frequent than glottic
cancer.
Nodal metastases occur early, upper and middle
jugular nodes are often involved.
Bilateral metastases may be seen in cases of epiglottic
cancer.
Pain on swallowing is the most frequent initial
symptom.
Mass in the neck may be the first sign.
Hoarseness is a late symptom.
Pain may be referred to ear by way of vagus nerve
and auricular nerve of Arnold
2.Glottic Cancer:
It isMC site of CA Larynx.
Mostly originates fromfree edge and upper surface
of anterior l/3rd of true vocal cord followed by
middle third l/3rd.
Spread locally to anterior commissure than to
opposite cord(conus elasticus initially acts as
barrier for subglottic spread).
Fixation of vocal cord indicates spread of disease to
thyroaretenoid muscle and is a bad prognostic sign.
Asvocal cord is free of lymphatics, nodal
metastases is never seen in cordal
Cancer, unless the disease spread beyond
membranous cord.
Hoarseness is MC and the earliest symptom
because of this glottic cancer is detected early.
Subglottic Cancer:
Least common site.
Subglottic region extends from glottic area to lower
border of cricoid cartilage.
Spreads locally around anterior wall to opposite side
or down wards to trachea, upward spread to vocal cord
is late and hoarseness is not an early symptom.
Lymphatic metastases to prelaryngeal, pretracheal
(Delphian node) and lower jugular nodes
Earliest and most prominent symptom is stridor,
but it appears only in advanced stage. Hoarseness is
late feature.

199. Answer is A (Anterior nasal cavity): Internet
Thudichum's nasal speculum is used to visualize Anterior
nasal cavity.
Thudichum's nasal speculum
Functions
Used for visualization of external nose
- antero-inferior part of nasal septum
- anterior part of inferior & middle
turbinate


200. Answer is D (Glomus jugular tumour): Pulsatile
Tinnitus in ear is due to Glomus jugular tumour.
GlomusTumour
MC benign neoplasm of middle ear, origin
from glomus bodies
Glomus bodies resemble carotid body in
structure and are found in dome of jugular bulb or
on the promontory along the course of tympanic
branch of IXth cranial nerve (Jacobson's
nerve)
Tumour consists of paraganglionic cells derived
from the neural crest
Generally seen in the middle age, females are
affected five times more

Pathology :
Benign, non-encapsulated, extremely
vascular neoplasm
Rate of growth is very slow and is locally
invasive
Abundance ofthin walled blood sinusoids with
no contractile muscle coat, accounting
forprofuse bleeding
May invade jugular foramen and the base of the
skull, causingIXth to Xllth cranial nerve palsies
Clinical Features:
When tumour is intra-tympanic:
Earliest symptoms are deafness (conductive &
slowly progressive) and tinnitus (pulsatile & of
swishing character, synchronous with pulse,
temporarily stopped by carotid pressure)
Otoscopy shows a red reflex through intact
tympanic membrane.
"Rising sun" appearance is seen when tumor
arises from the floor of middle ear.
"Pulsatile sign" (Brown's sign) is positive, i.e.
when ear canal pressure is raised with Siegle's
speculum, tumor pulsates vigorously and then
blanches; reverse happens with release of
pressure.
When tumor presents as a polyp:
Profuse bleeding from the ear either
spontaneously or an attempts to clean it
Examination reveals a red, vascular polyp filling
the meatus
Phelp's sign: Absence of normal crest of bone
between carotid canal and jugular fossa on lateral
tomography is virtually diagnostic of glomus
jugulare tumor
Treatment:
Surgical removal, radiation, embolization or
combination

S-ar putea să vă placă și